Spine Flashcards

1
Q

(Ped) 9. A 4-year-old girl with low-grade fever and back pain has an elevated Erythrocyte Sedimentation Rate (ESR) but normal White Cell Count (WCC). Diskitis is considered. Which is the best answer regarding diskitis?

A. Usually affects children 4-10 years old

B. Thoracic spine is most commonly affected

C. Usually involves 3 consecutive disc spaces

D. Decreased marrow intensity on 2 consecutive vertebrae on T1 MR is characteristic

E. Radiographs are usually positive before bone scan

A

D. Decreased marrow intensity on 2 consecutive vertebrae on T1 MR is characteristic

Diskitis is the most common paediatric spine pathology.

Staphylococcus aureus is the most common causative organism.

The peak ages are 6 months-4 years and 10-14 years.

L3-4 and L 4-5 are most commonly affected sites.

How well did you know this?
1
Not at all
2
3
4
5
Perfectly
2
Q

(Ped) 10. An eccentric mass expands the cord on MR of a 5-year-old boy. The mass is isointense on T1 and hyperintense on T2. Which is the most common intramedullary neoplasm in children, exhibiting these features?

A. Ependymoma

B. Astrocytoma

C. Ganglioglioma

D. Haemangioblastoma

E. Subependymoma

A

B. Astrocytoma

Astrocytomas appear as homogenously ill-defined cord tumours, with poorly defined margins and patchy irregular enhancement with gad. Masses can take form of eccentric irregular tumour cysts, polar cysts & syrinxes

How well did you know this?
1
Not at all
2
3
4
5
Perfectly
3
Q

@# 28. A follow-up of a young man under the care of the neurooncologists reveals a drop in metastases. Which statement is most true?

A. Drop metastases tend to be in the upper spine

B. Metastases within the spinal canal are usually ventral

C. Glioblastoma is the commonest cause of drop metastases

D. Are associated with positive CSF cytology in approximately 10% of cases

E. Usually demonstrates homogenous enhancement with contrast

A

E. Usually demonstrates homogenous enhancement with contrast

Drop metastases are usually dorsal in location in the spinal canal.

Medulloblastomas are the most common cause of drop metastases.

A higher percentage of CSF cytology is positive.

How well did you know this?
1
Not at all
2
3
4
5
Perfectly
4
Q

(MSK) 30. A 16-year-old girl who has recently moved from India to the UK has back pain. Considering tuberculous spondylitis:

A. 10% of skeletal TB involves the spine

B. Infection usually begins in the posterior part of the vertebral body

C. Medial bowing of the psoas shadow on plain film may indicate an abscess

D. The upper thoracic spine is most commonly affected

E. Calcification within a psoas abscess is highly likely to represent TB

A

E. Calcification within a psoas abscess is highly likely to represent TB

50% of skeletal TB involves the spine, with the lower thoracic and upper lumbar regions being most commonly affected.

It usually begins at the anterior vertebral body.

A psoas abscess may cause lateral bowing of the psoas shadow on plain film.

How well did you know this?
1
Not at all
2
3
4
5
Perfectly
5
Q

@# 42. Which is a cause of solitary dense pedicle rather than erosion/absence?

A. Osteoblastoma

B. Metastatic carcinoma

C. Neurofibroma

D. Tuberculosis with paravertebral abscess

E. Aneurysmal bone cyst

A
  1. A Osteoblastoma is a cause of a solitary dense pedicle
How well did you know this?
1
Not at all
2
3
4
5
Perfectly
6
Q
  1. Which is most likely to represent an intramedullary mass lesion?

A. Ependymoma

B. Meningioma

C. Neurofibroma

D. Arachnoid cyst

E. Abscess

A

A. Ependymoma

Other intramedullary masses include astrocytomas are dermoids (lipoma/teratomas), acutely expanding infarcts and haematoma.

How well did you know this?
1
Not at all
2
3
4
5
Perfectly
7
Q
  1. Which is the most common cause of erosion of the odontoid peg?

A. Rheumatoid arthritis

B. SLE

C. Ankylosing spondylitis

D. Psoriasis

E. Down’s syndrome

A

A. Rheumatoid arthritis

Rheumatoid arthritis is the most common cause of erosion of the odontoid peg.

How well did you know this?
1
Not at all
2
3
4
5
Perfectly
8
Q

(MSK) 47. On review of a casualty film of a patient involved in a road traffic accident, which of the following is an unstable cervical spine fracture?

A. Unilateral facet dislocation

B. Clay shovellers

C. Jefferson

D. Burst

E. Extension tear drop

A

C. Jefferson

Unstable fractures include bilateral facet dislocation, flexion teardrop, hangman’s, hyperextension dislocation, Jefferson, odontoid and atlanto-occipital dislocation

How well did you know this?
1
Not at all
2
3
4
5
Perfectly
9
Q

(Ped) 2) A 14-year-old boy who is a keen gymnast and fast bowler gives a history of several months of central low back pain that suddenly worsened during a game of cricket when he also developed bilateral shooting pains in his legs. There is no overt deformity on clinical examination, but lower back tenderness with generally restricted movement is noted. He undergoes radiographic, CT and MR imaging. What is the most likely radiological finding to explain the patient’s current symptoms?

a. herniated intervertebral disc

b. discitis

c. Scheuermann’s disease

d. spondylolysis

e. spondylolisthesis

A

e. spondylolisthesis

Back pain in adults is common and most frequently non-specific.

In contrast, back pain in children is less common and often caused by a serious underlying condition.

Spondylolysis is a defect in the pars interarticularis, the weakest part of the vertebra, and is an acquired condition even in childhood, where it is usually due to repetitive microtrauma in athletically active children.

In isolation, it does not cause neurological symptoms, but bilateral defects can allow slippage of one vertebra over another, creating an abnormality of alignment, a spondylolisthesis.

Disc herniation in children is rare and occurs as a result of a traumatic event rather than degeneration. It is lateral.

Scheuermann’s disease is associated with a kyphotic deformity.

How well did you know this?
1
Not at all
2
3
4
5
Perfectly
10
Q

(MSK) 4) CT of the cervical spine is performed on an intubated emergency patient who was a restrained driver in a high-speed motor vehicle collision. This reveals bilateral C2 pedicle fractures. What is the most likely underlying mechanism of injury?

a. hyperflexion and rotation

b. hyperextension followed by hyperflexion

c. axial loading

d. hyperextension and traction

e. hyper-rotation

A

d. hyperextension and traction

The fracture described is a hangman fracture. This involves either the pedicles or pars interarticularis of C2 bilaterally.

The mechanism is usually extension and traction (as caused during hanging).

Hyperflexion injuries produce anterior tear-drop or of a vertebral body wedge fractures.

Axial loading can produce a burst fracture of C1 (Jefferson’s fracture) or a vertebral body elsewhere in the spine.

Hyperflexion and extension are associated with longitudinal ligament injury.

Hyperrotation is associated with soft-tissue injury or facet joint dislocation.

How well did you know this?
1
Not at all
2
3
4
5
Perfectly
11
Q

(MSK) 7) A child passenger is admitted to accident and emergency following a road traffic collision. Radiographs of the spine show a horizontal fracture involving the vertebral body and pedicles of L2. Associated injury to which of the following abdominal organs is most likely?

a. duodenum

b. pancreas

c. spleen

d. liver

e. rectum

A

a. duodenum

The spinal injury described is a Chance fracture, a fracture through the vertebral body and pedicles caused by hyperflexion, therefore causing compression of the spine anteriorly and distraction posteriorly.

This injury typically occurs in back-seat passengers wearing a lap seat belt during a road traffic collision.

In children, there is a 50% incidence of associated intra-abdominal organ injury.

Retroperitoneal organs are most vulnerable, being closest to the spine. Duodenal injuries are most common, and have a significant associated mortality. The pancreas is also commonly injured due to its retroperitoneal location

How well did you know this?
1
Not at all
2
3
4
5
Perfectly
12
Q

11) Degenerative spinal vertebral body endplate changes, as seen on MRI, may have which of the following appearances?

a. type I – high T1W and low T2W signal

b. type I – low T1W and low T2W signal

c. type II – high T1W and high T2W signal

d. type II – low T1W and high T2W signal

e. type II – high T1W and low T2W signal

A

c. type II – high T1W and high T2W signal

The endplates in degenerative disc disease have three described appearances during their evolution, which are also known as Modic changes.

Type I (marrow oedema) changes show low signal on T1W and high signal on T2W sequences.

Type II (fatty marrow) changes show high signal on both T1Wand T2W sequences.

Type III (sclerosis) changes are low signal on both T1W and T2W sequences.

How well did you know this?
1
Not at all
2
3
4
5
Perfectly
13
Q

(MSK) 11) You receive a referral while on call from the orthopaedic consultant regarding a middle-aged woman with a long history of simple back pain. She has attended accident and emergency complaining of worsening lower lumbar pain with a several-hour history of progressive urinary retention, faecal incontinence, saddle anaesthesia and mild bilateral leg weakness. Which method of imaging would you recommend as most appropriate?

a. plain radiography

b. myelogram

c. CT

d. CT myelogram

e. MRI

A

e. MRI

Bilateral lower limb involvement suggests a myelopathy rather than a radiculopathy. The presence of urinary and bowel symptoms and saddle anaesthesia suggests compression of lumbosacral nerve roots. This complex of symptoms is cauda equina syndrome and is considered an orthopaedic emergency because of the likelihood of permanent neurological impairment, particularly affecting the autonomic supply to the bladder or bowel, which can result in permanent incontinence if surgery is delayed. The Royal College of Radiologists recommends proceeding straight to MRI in patients who have ‘red flag’ signs.

How well did you know this?
1
Not at all
2
3
4
5
Perfectly
14
Q

(Ped) 15) Of the following findings on a cervical spine radiograph in a 10- year-old child, which is abnormal in the context of a traumatic injury?

a. anterior wedging of the C3 vertebral body

b. anterolisthesis in flexion at C2–3

c. prevertebral soft-tissue thickness of 15 mm at C6

d. predental space of 6 mm in flexion

e. predental space of 3 mm in neutral

A

d. predental space of 6 mm in flexion

The maximum predental space is 2.5–3 mm in an adult and 5 mm in a child. Any widening suggests injury to the alar ligamentous complex in the context of trauma.

Other causes of widening are Down’s syndrome, rheumatoid arthritis, neurofibromatosis and osteogenesis imperfecta.

Anterior wedging of C3 and pseudosubluxation at C2–3 and C3–4 are within normal limits in children.

Additionally, prevertebral soft tissues can be greater than in the adult, certainly up to 100% of the anteroposterior dimension of the vertebral body at the C6 level.

How well did you know this?
1
Not at all
2
3
4
5
Perfectly
15
Q

16) A 46-year-old female presents with back pain and increasing weakness of the lower limbs. An MR scan shows a lesion in the cord at the level of T11. Which of the following features would suggest an ependymoma rather than demyelination?

a. multiple lesions

b. expansion of the cord

c. high signal on T2W images

d. enhancement with gadolinium

e. peripheral low signal on all sequences

A

b. expansion of the cord

Ependymomas are the commonest tumour of the spinal cord in adults, accounting for 40–60% of cord tumours. They present with a long history of pain, and sensory or motor disturbance. Less commonly, bladder and bowel dysfunction may occur. Expansion of the cord is more often seen with ependymomas than with demyelination.

Both lesions may enhance and have high signal on T2W images, but multiplicity is more often seen with demyelination.

Peripheral low signal, usually indicating haemosiderin, is not a feature of either of these lesions.

How well did you know this?
1
Not at all
2
3
4
5
Perfectly
16
Q

(MSK) 18) In a patient who presents with acute femoral nerve radiculopathy, which of the following MRI sequences is the most useful in the diagnosis of a far lateral upper lumbar vertebral disc protrusion?

a. sagittal STIR

b. sagittal T1

c. sagittal T2

d. axial STIR

e. axial T2

A

e. axial T2

The far lateral disc protrusion is the least common type of symptomatic disc herniation. It distinguishes itself from the posterolateral herniation in that the disc ruptures outside the spinal canal, lateral to the root foramen. The disc, instead of tethering the traversing nerve root, compresses the more rostral nerve root that has already exited the root foramen. The neurological symptoms therefore correspond to a lesion at the upper disc level, often leading to confusion in the diagnosis. It is also difficult to diagnose radiologically, as the far lateral location isusually not detected on the sagittal images but only on axial images. STIR is an inversion recovery sequence that suppresses fat and so highlights areas of increased fluid. However, it is not sensitive when the herniation is outside the fluid-filled spinal canal; therefore, the T2W gradient echo sequence is better at detecting far lateral disc herniation.

How well did you know this?
1
Not at all
2
3
4
5
Perfectly
17
Q

(MSK) 26) On plain radiographs of the neck in a 60-year-old man, which feature is most likely to support a diagnosis of diffuse idiopathic skeletal hyperostosis rather than ankylosing spondylitis?

a. enthesopathy

b. confluent intervertebral bony bridging

c. sparing of the posterior elements

d. sparing of the sacroiliac joints

e. changes limited to the thoracic spine

A

d. sparing of the sacroiliac joints

Diffuse idiopathic skeletal hyperostosis (DISH) is an ankylosing disorder of the spine.

It is most commonly seen in the thoracic region but may involve cervical and lumbar regions.

Diagnostic criteria are of flowing calcification along the anterolateral border of at least four vertebral bodies, relative preservation of intervertebral disc height, and absence of sacroiliac joint or apophyseal involvement.

These three criteria aid differentiation of spondylosis deformans, intervertebral osteochondromatosis and ankylosing spondylitis respectively.

Extra-spinal manifestations of DISH include Achilles tendinosis, tennis elbow, calcaneal and olecranon enthesopathy and dysphagia.

Whiskering is seen radiographically at tendinous insertions, particularly of the pelvis.

How well did you know this?
1
Not at all
2
3
4
5
Perfectly
18
Q

(MSK) 27) A lumbar spinal MRI is performed on a young man of south-east Asian origin for back pain and pyrexia of unknown origin. It reveals an anterior paraspinal soft-tissue mass at levels L1 to L3 centred at the L2–3 intervertebral disc. It is located deep to and displaces the anterior longitudinal ligament, and extends into the left psoas muscle. The mass returns intermediate signal on T1W images and high signal on T2W images. There are oedematous changes in the adjacent vertebral bodies, but the intervertebral discs are spared. What is the most likely infectious organism?

a. Mycobacterium tuberculosis

b. Actinomyces

c. HIV

d. Staphylococcus aureus

e. Aspergillus fumigatus

A

a. Mycobacterium tuberculosis

The musculoskeletal system is affected in only 1–3% of tuberculous infections, but the spine is the most common skeletal location affected, accounting for 50% of musculoskeletal tuberculosis.

Tuberculous spondylitis (or Pott’s disease) can result in significant neurological sequelae.

A history of pulmonary infection may or may not be present.

The infection usually begins in the anterior vertebral body via haematogenous spread.

The intervertebral discs are frequently involved, and the loose internal structure of the disc allows the infection to disseminate more widely, often resulting in paraspinal or psoas abscess.

Calcification within the abscess is very specific for tuberculosis.

The disease process often leads to vertebral collapse with gibbous deformity and obliteration of the disc space. However, elevation of the anterior ligaments by subligamentous abscess allows tracking superiorly and inferiorly, and classically spares the disc.

Tuberculosis characteristically results in little reactive sclerosis or periosteal reaction, which helps to distinguish it from pyogenic infections.

How well did you know this?
1
Not at all
2
3
4
5
Perfectly
19
Q

(MSK) 31) Plain radiographs of the spine in a 40-year-old man performed following a road traffic collision reveal a slightly expanded midthoracic vertebral body with coarse vertical trabeculations. Subsequent CT shows a ‘polka-dot’ appearance to the same vertebral body in the axial plane. What is the most likely disorder affecting the vertebra?

a. aneurysmal bone cyst

b. osteoid osteoma

c. haemangioma

d. compression fracture

e. osteopoikilosis

A

c. haemangioma

Metastatic disease, myeloma and lymphoma are the most common malignant spinal tumours, and haemangioma is the most common benign tumour of the spine.

The appearances described are characteristic of a vertebral haemangioma. On MRI, these lesions typically appear of mottled low-to-high signal on T1W images depending on the degree of fat present, and of very high signal on T2W images.

Other primary osseous lesions of the spine are more unusual but may exhibit characteristic imaging features that can help develop a differential diagnosis.

Radiological evaluation of a patient who presents with osseous vertebral lesions often includes radiography, CT and MRI. The complex anatomy of the vertebrae means that CT is more useful than conventional radiography for evaluating lesion location and assessing bone destruction.

The diagnosis of spinal tumours is based on patient age, topographic features of the tumour and lesion pattern as seen on imaging.

How well did you know this?
1
Not at all
2
3
4
5
Perfectly
20
Q

45) A 21 year old presents with back pain, increasing over time. There are no neurological symptoms. A radiograph of the lumbar spine shows a grade II spondylolisthesis at L5–S1. Which of the following features would suggest the presence of bilateral spondylolysis as the cause?

a. narrowing of AP diameter of spinal canal at L5–S1

b. widening of AP diameter of spinal canal at L5–S1

c. lucencies through the laminae of L5

d. sclerosis of the pedicles of L5

e. reduced height of the L5–S1 disc

A

b. widening of AP diameter of spinal canal at L5–S1

Spondylolysis (pars defect) is seen in 3–7% of the population, with 50% being symptomatic.

L5 is the most commonly affected level.

There are fractures through the pars interarticularis, which may be unilateral or bilateral.

Spondylolisthesis can occur only when pars defects are bilateral.

The AP diameter of the canal is widened, as the vertebral body and pedicles are detached from the posterior elements and migrate anteriorly.

Narrowing of the canal is seen with other causes of spondylolisthesis, particularly degenerative causes.

Sclerosis of one pedicle is seen with unilateral pars defects, as the contralateral pedicle undergoes reactive sclerosis due to excessive stress.

How well did you know this?
1
Not at all
2
3
4
5
Perfectly
21
Q

(MSK) 36) A 40-year-old man falls down the stairs and remains unconscious for several hours. On admission to hospital, he is found to have bilateral upper limb weakness, patchy sensory loss, full power in the lower limbs and a normal level of consciousness. Plain radiographs of the cervical spine and CT of the brain are normal. On MRI of the cervical spine, there is a small area of oedema identified within the cord. Clinical symptoms persist for 4 days following injury. What is the most likely diagnosis?

a. central cord syndrome

b. anterior cord syndrome

c. SCIWORA (spinal cord injury without radiological abnormality)

d. spinal shock

e. Brown-Se’quard syndrome

A

a. central cord syndrome

In trauma, an incomplete spinal cord injury is one in which there is any degree of sparing of motor or sensory function distal to the site of injury, whereas complete cord injury results in complete lack of neurological function distal to the injury.

The diagnosis can be made only in the absence of spinal shock, a transient spinal cord concussion.

Central cord syndrome is the most common incomplete injury, and is associated with hyperextension injury in middle-aged patients; injury to centrally located grey matter in the cord causes a greater motor neurological deficit in the upper than in the lower extremities. Sensory involvement can be variable, and bowel and bladder function may be affected.

Anterior cord syndrome, caused by anterior spinal vascular insufficiency, causes complete motor paralysis with sparing of the posterior columns.

SCIWORA is seen in children, when the elastic cervical spine deforms sufficiently to cause cord injury but without any radiological findings.

Brown-Se´quard syndrome results from hemitransection and causes ipsilateral muscle paralysis and contralateral hyperaesthesia to pain and temperature.

How well did you know this?
1
Not at all
2
3
4
5
Perfectly
22
Q

(MSK) 43) A middle-aged male patient who has previously undergone partial discectomy for radiculopathy, has a lumbar spine MRI due to a recurrence of his symptoms. T1W images show a low-signal area of tissue contiguous with the previously operated intervertebral disc and impinging upon the adjacent exiting nerve root. Which single additional finding favours a diagnosis of postoperative fibrosis over recurrent disc protrusion?

a. high signal on STIR sequence

b. enhancement with intravenous gadolinium

c. evolution at 6-month serial imaging

d. oedema in the surrounding bone

e. low signal on T2W images

A

b. enhancement with intravenous gadolinium

In MRI of the spine in postoperative discectomy patients with recurrent or persistent radiculopathy, a T1W sequence with intravenous gadolinium enhancement is added to distinguish between postoperative epidural fibrosis (or scarring) and recurrent disc herniation.

Both can have similar, low-signal appearances on unenhanced T1W and T2W images, but fibrosis will show enhancement with gadolinium whereas recurrent disc prolapse will not.

Difficulties arise where both conditions exist concurrently, and fibrosis that is not causing nerve root irritation may also enhance.

The importance of distinguishing between the two is that surgical treatment is indicated for recurrent disc herniation but is of no value in treating postoperative fibrosis, also known as failed back syndrome.

How well did you know this?
1
Not at all
2
3
4
5
Perfectly
23
Q

(MSK) 37) Vertebral sclerosis confined to the upper and lower endplates with preservation of the intervertebral disc space (‘rugger jersey spine’), is seen most commonly with which underlying condition?

a. osteoporosis

b. discitis

c. mucopolysaccharidosis

d. Paget’s disease

e. renal osteodystrophy

A

e. renal osteodystrophy

The ‘rugger jersey spine’ appearance refers to sclerotic bands along the superior and inferior endplates of the thoracic and lumbar vertebral bodies.

These bands represent accumulation of excess osteoid and result in a striped appearance of the vertebral bodies.

Despite being poorly mineralized, the accumulated osteoid appears opaque on plain radiographs because of its increased volume compared with that of normal bone.

The ‘rugger jersey spine’ is said to be almost pathognomonic for the osteosclerosis seen with the secondary hyperparathyroidism of chronic renal failure.

Renal osteodystrophy is a term for the constellation of musculoskeletal abnormalities occurring with chronic renal failure.

Osteoporosis and Paget’s disease are more likely to affect the whole of the vertebrae diffusely.

Discitis usually causes a reduction in the intervertebral disc space on radiographs, with indistinct endplates.

The mucopolysaccharidoses result in anterior vertebral body beaking rather than sclerosis.

How well did you know this?
1
Not at all
2
3
4
5
Perfectly
24
Q

59) A 47-year-old female presents with gradual onset back pain over 4 weeks, with associated pyrexia and tenderness at the thoracolumbar junction. Radiographs show destruction of the endplates at the T12–L1 disc level. Which of the following features on further imaging would suggest tuberculous over pyogenic discitis as a cause?

a. single-level involvement

b. paravertebral, soft-tissue mass

c. epidural abscess

d. disc-space loss

e. calcification

A

e. calcification

Tuberculous discitis tends to occur in children and adults around the age of 50 years.

It most commonly affects T12–L1, compared with pyogenic discitis, which tends to occur more distally.

Tuberculous discitis often affects more than one level contiguously.

Paravertebral masses and epidural abscesses are seen as complications in all types of discitis, but calcification within an abscess is virtually diagnostic for tuberculosis.

Disc-space loss is also seen in all types of discitis, although it tends to be better preserved with tuberculous infection.

How well did you know this?
1
Not at all
2
3
4
5
Perfectly
25
Q

(Ped) 63) A 12-year-old presents with increasing back pain over 2 weeks associated with malaise. He has mild pyrexia. Plain films of the lumbar spine show reduced disc space between the second and third lumbar vertebrae with loss of clarity of the endplates. What investigation would be most likely to help make the diagnosis?

a. CT

b. MRI

c. bone scan

d. labelled white cell scan

e. gallium scan

A

b. MRI

Discitis is the commonest paediatric spinal disease. It is secondary to bacterial invasion of the disc through the endplate. Plain films typically show reduced disc-space height and loss of clarity of the endplates in the acute phase. MRI is the best investigation, as it is the most sensitive. Reduced T1 signal is seen in the adjacent marrow due to oedema, with initially variable, then increased, T2 signal. Complications such as epidural abscess are well demonstrated. CTwill show the endplate changes and any paravertebral inflammatory mass. Bone scans and white cell scans have much poorer sensitivity than MRI, though this is improved with the use of SPECT.

How well did you know this?
1
Not at all
2
3
4
5
Perfectly
26
Q

(MSK) 67) In reviewing a fracture of the spine at the thoracolumbar junction in a major trauma case, which single indicator on CT is most sensitive for inferring instability?

a. widened facet joints

b. two-column malalignment

c. soft-tissue swelling

d. rotational abnormality

e. increased intervertebral disc space

A

b. two-column malalignment

The spine can be divided anatomically into three columns:

the anterior column contains the anterior longitudinal ligament, anterior half of the vertebral body and anterior annulus fibrosus;

the middle column contains the posterior half of the vertebral body, posterior longitudinal ligament and the posterior annulus fibrosus;

and the posterior column contains the posterior elements of the spine, facet joint capsule and interspinous ligaments.

Two intact columns are required for intrinsic spinal stability.

Disruption of two columns can therefore be used to infer instability.

Usual traumatic patterns are anterior and middle, or posterior and middle, disruption.

Isolated middle column interruption can occur after trauma or surgery, or as a congenital abnormality, and is also considered potentially unstable.

How well did you know this?
1
Not at all
2
3
4
5
Perfectly
27
Q

74) A 65-year-old man presents with back stiffness and painful hips. Radiographs of the thoracolumbar spine and pelvis show ossification of the iliolumbar and sacroiliac ligaments with whiskering of the ischial tuberosities. Which of the following additional features is most likely to be seen on the spine radiographs?

a. flowing osteophytes over several vertebral levels

b. squaring of vertebral bodies

c. reduced disc spaces

d. sclerosis of vertebral bodies

e. posterior longitudinal ligament calcification

A

a. flowing osteophytes over several vertebral levels

The pelvic features are suggestive of diffuse idiopathic skeletal hyperostosis (DISH). Flowing anterior vertebral osteophytes, especially in the lower thoracic region, are very suggestive of this condition, and disc spaces are usually well preserved.

Vertebral body squaring is seen in a number of conditions, including ankylosing spondylitis, but not DISH.

How well did you know this?
1
Not at all
2
3
4
5
Perfectly
28
Q

(MSK) 76) A 50-year-old mechanic with a long history of back pain presents to the spinal clinic complaining of sudden onset of numbness and pain over the right lateral calf and dorsum and sole of the right foot following heavy lifting. Which of the following spinal pathologies is most likely to explain the patient’s symptoms?

a. lumbar spinal stenosis

b. paracentral L4–5 disc protrusion

c. paracentral L5–S1 disc protrusion

d. far lateral L4–5 disc protrusion

e. central L5–S1 disc protrusion

A

b. paracentral L4–5 disc protrusion

Degenerative disc disease of the spine is one of the leading causes of functional incapacity and chronic disability in the working population, affecting both men and women.

Although there is no universally established nomenclature for describing disc herniation,

‘protrusion’ is commonly used if the herniation is broader than it is deep

and ‘extrusion’ if it is deeper than it is broad.

A disc ‘bulge’ is used to describe a herniation that is very broad based and may even be circumferential, with a generalised disc bulge being one that affects at least half of the periphery. As a result of the strong posterior longitudinal ligament, posterior disc herniation is often paracentral, i.e. to the side of the midline. This can result in compression of the transiting nerve root in the lateral recess, which is the one that will exit at the level below.

A lateral disc herniation narrowing the neural foramen compresses the exiting nerve root. Therefore, for a given intervertebral disc, a paracentral herniation will affect the nerve that exits one level below, whereas a lateral protrusion affects the nerve root at that level.

How well did you know this?
1
Not at all
2
3
4
5
Perfectly
29
Q

(MSK) 81) Which of the following is not an appropriate indication for percutaneous polymethylmethacrylate cement vertebroplasty?

a. progressive osteoporotic deformity

b. painful osteoporotic collapse

c. painful haemangioma

d. painful osteoid osteoma

e. painful metastases

A

d. painful osteoid osteoma

Percutaneous cement vertebroplasty is a treatment for vertebral compression fractures that involves the injection of acrylic bone cement into the vertebral body in order to relieve pain, stabilize fractured vertebrae or, in some cases, restore vertebral height. Current guidelines from the National Institute for Health and Clinical Excellence (NICE), regarding the use of vertebroplasty in the UK, state that it may be used for pain relief in patients with severe painful osteoporosis with loss of height, compression fractures of the vertebral body, symptomatic vertebral haemangioma and painful vertebral body tumours (metastases or myeloma). Review of current evidence indicates some level of pain relief in 58–97% of patients.

How well did you know this?
1
Not at all
2
3
4
5
Perfectly
30
Q

(MSK) 82) On MRI of the spine demonstrating vertebral body collapse, which additional feature favours an underlying diagnosis of malignancy rather than osteoporosis?

a. bone fragment retropulsion

b. focal low signal in the vertebral body on T1W images

c. diffuse intermediate signal in the vertebral body on T2W images

d. no enhancement with gadolinium

e. convex posterior border to the vertebral body

A

e. convex posterior border to the vertebral body

A convex bulge involving the whole posterior border of the vertebral body strongly suggests vertebral body expansion by tumour invasion, and is only very rarely a feature of osteoporosis. Other findings on MRI suggestive of malignancy include a soft-tissue mass, involvement of the pedicles, and heterogeneous high signal on T1W post-contrast or T2W images.

Retropulsion of bone fragments, focal T1 low signal or an isointense appearance on T1W or T2W images suggests osteoporotic collapse.

How well did you know this?
1
Not at all
2
3
4
5
Perfectly
31
Q

(MSK) 88) On radiographs and MRI of the spine performed for lower back pain with clinical signs of radiculopathy, which of the following features favours a diagnosis of discitis rather than degenerative disc disease?

a. vacuum phenomenon in the discs

b. reduced disc space

c. intermediate signal posterior to the vertebral body on T1W images

d. vertebral endplate low signal on T1W images

e. Schmorl’s nodes

A

c. intermediate signal posterior to the vertebral body on T1W images

Intermediate signal in the extradural space on T1W images is the most common appearance of extradural abscess formation.

The most common primary focus of infection is discitis, but abscess formation may also be spontaneous. Patients particularly at risk are those with a history of diabetes mellitus, intravenous drug use, trauma, haemodialysis or recent surgery (particularly dental).

MRI features of extradural abscess include iso- or slight hyperintensity on T1W images when compared with the spinal cord.

High signal on T2W and proton density sequences makes it difficult to differentiate abscess from CSF, but these sequences are useful, as osteomyelitis and paravertebral abscess are well visualized as high-signal lesions.

Administration of intravenous gadolinium contrast characteristically demonstrates diffuse enhancement of the solid component of the abscess.

How well did you know this?
1
Not at all
2
3
4
5
Perfectly
32
Q

(MSK) 92) Vertebral bone marrow oedema, seen as low signal on T1W and high signal on T2W MR images, occurs typically in all but which of the following conditions?

a. degenerative disc disease

b. multiple myeloma

c. osteoporotic collapse

d. spondylolysis

e. ankylosing spondylitis

A

b. multiple myeloma

Multiple myeloma is a malignant condition of plasma cells that commonly shows infiltration of the bone marrow, best seen on MRI.

Patterns of infiltration can be classified as focal, diffuse or variegated.

Although marrow infiltration returns similar signal to marrow oedema on T1W and T2W images, infiltration will show diffuse enhancement following administration of intravenous gadolinium.

The pattern of infiltration also differs. Infiltration will be patchy and randomly distributed throughout the vertebral bone.

In contrast, bone oedema occurs adjacent to its cause, being linear at the endplates in the case of degenerative disc disease, and linear with a fracture line in osteoporotic collapse, in the pedicles adjacent to spondylolysis or at the entheses in ankylosing spondylitis.

How well did you know this?
1
Not at all
2
3
4
5
Perfectly
33
Q

(Ped) 92) An 8-year-old girl presents with back pain. Clinically, there is a double-curve scoliosis convex to the left in the thoracic region and to the right in the lumbar region. There is no focal neurology. MRI shows the conus behind the L3 vertebral body, and the filum terminale is 3 mm in thickness at the L5-S1 level. A high-signal lesion is seen on T1W and T2W images in the canal behind the L5 and S1 vertebral bodies. What is the most likely diagnosis?

a. diastematomyelia

b. meningocele

c. syringomyelia

d. tethered cord

e. developmental scoliosis

A

d. tethered cord

Tethering of the cord results in the conus lying lower than normal and is associated with scoliosis, thickening of the filum terminale (>2 mm at the L5-S1 level on axial T1 image) and spinal lipoma.

Less frequent associations are Chiari malformations, syrinx, myelomeningocele, diastematomyelia and dermal sinus.

Diastematomyelia is a midline sagittal cleft in the cord, often with a bony/fibrous septum.

Syringomyelia is dissection of cerebrospinal fluid through the cord, producing high T2 signal within the cord. This is associated with several neurological abnormalities.

Developmental scoliosis occurs in adolescent girls, is convex to the right and has no associated neurological symptoms.

How well did you know this?
1
Not at all
2
3
4
5
Perfectly
34
Q

(MSK) 94) A 65-year-old man undergoes radiographs of the lumbar spine and pelvis for lower back pain. A destructive lytic lesion is identified in the midline of the inferior sacrum with internal areas of calcification. Subsequent MRI reveals a heterogeneous lesion replacing much of the sacrum, which returns moderate low signal on T1W and high signal on T2W images, with a soft-tissue component extending into the presacral soft tissues. The lesion shows patchy moderate enhancement with intravenous gadolinium. What is the most likely diagnosis?

a. metastasis

b. giant cell tumour

c. aneurysmal bone cyst

d. chordoma

e. plasmacytoma

A

d. chordoma

Chordomas arise from notochordal rests and therefore almost always occur in the midline. They are the most common primary malignant sacral tumour and account for 2–4% of all malignant tumours of bone. They are found at all ages but most commonly occur in the fourth to seventh decades of life. Approximately half develop in the sacrococcygeal region. There is usually a large soft-tissue component and the tumour may extend across the intervertebral disc space or sacroiliac joint. Overall, the most common sacral lesion is metastasis due to the high red marrow content, but other primary malignant lesions include myeloma, Ewing’s sarcoma and lymphoma. The most commonly found benign tumours are giant cell tumours and aneurysmal bone cysts. Despite being relatively common in the rest of the spine, haemangiomas and osteoid osteomas are rare.

How well did you know this?
1
Not at all
2
3
4
5
Perfectly
35
Q

10 A patient presents with bladder dysfunction and leg weakness. MRI of the spine reveals a solitary intradural, extramedullary tumour of the thoracic spinal cord. It is posteriorly located and is of high signal intensity of T2W.
What is the most likely diagnosis?

(a) Nerve sheath tumour

(b) Leptomeningeal metastasis

(c) Chordoma

(d) Meningioma

(e) Astrocytoma

A

(d) Meningioma

Of the listed options, only nerve sheath tumours, meningiomas and leptomeningeal metastasis are typically intradural and extramedullary (chordomas are extradural and astrocytomas are intramedullary). Although nerve sheath tumours are more common, meningiomas are more likely if the tumour is solitary, posterior and does not contain areas of low signal intensity on T2W

How well did you know this?
1
Not at all
2
3
4
5
Perfectly
36
Q

(MSK) 17 You are asked to review a series of plain films of the cervical spine of an adult patient. Which of the following is abnormal?

(a) On the lateral view, the distance between the anterior arch of C1 and the anterior aspect of the odontoid peg is 2 mm

(b) On the lateral view, the soft tissues anterior to C2 are 9 mm thick

(c) Harris’ white ring is incomplete in its inferior aspect

(d) On the lateral view, the C4-5 interspinous distance is 30% greater than the CS-6 interspinous distance

(e) On the lateral view, the soft tissues anterior to C6 are 20 mm thick

A

(b) On the lateral view, the soft tissues anterior to C2 are 9 mm thick

The distance between the anterior arch of C1 and the anterior aspect of the odontoid peg should be no more than 3 mm in an adult.

On the lateral view, the maximum width of the prevertebral soft tissues is: 7 mm at C1-4, 22 mm at C5-7.

Harris’ ring is often incompletE;i in its inferior aspect.

On the long AP view, no single interspinous distance should be more than 50% wider than the one immediately above or below it.

How well did you know this?
1
Not at all
2
3
4
5
Perfectly
37
Q

(Ped) 27 A 14 year old boy presents with mid/ lower back pain. He is noted to be kyphotic on examination. A lateral X-ray of the thoraco-lumbar vertebrae shows anterior wedging of the T8- 10 vertebral bodies. What is the most likely diagnosis?

(a) Ankylosing spondylitis

(b) Eosinophilic granuloma

(c) Morquio’s syndrome

(d) Osteogenesis imperfecta

(e) Scheuermann’s disease

A

(e) Scheuermann’s disease

Scheuermann’s is thought to be caused by herniation of disc material through congenital end-plate defects during the adolescent growth spurt (presents 13-17 yrs). It accounts for 31 % of cases of back pain in adolescent boys and is located in the thoracic (75%) or thoracolumbar spine (25%). At least one vertebral body needs to be involved with anterior wedging of >5° (usually 3-5 are involved), and there must be kyphosis of >35°. The posterior aspect of the vertebral body is protected by posterior articulation. The other listed conditions are potential causes of kyphosis in children.

How well did you know this?
1
Not at all
2
3
4
5
Perfectly
38
Q

@# 32 An MR of the spine in a neonate reveals two separate hemichords in two separate dural tubes. Which type of split cord malformation does this represent?

(a) Type I

(b) Type II

(c) Type Ill

(d) Type IV

(e) Type V

A

(a) Type I

This is a type I malformation, also known as diastematomyelia. A type II of malformation comprises two hemicords within a single dural tube, also known as diplomyelia. There is no type III, IV or V malformations.

How well did you know this?
1
Not at all
2
3
4
5
Perfectly
39
Q

(MSK) 37 An MRI examination of the lumbar spine demonstrates endplates with reduced signal intensity on T1W and increased signal intensity on T2W. What is the most appropriate diagnosis?

(a) Normal

(b) Type I Modic change

(c) Type II Modic change

(d) Type Ill ty1pdic change

(e) Type IV Modic change

A

(b) Type I Modic change

Modic degenerative changes are bone marrow and endplate changes adjacent to degenerative lumbar intervertebral discs; they are commonest at the L4-L5 and L5-S1 level. Modic II is more prevalent, but Modic I changes are more likely to be symptomatic. Type I changes have MRI appearances of fluid (low T1, high T2), type II changes have the characteristics of fat due to red marrow replacement (high T1 and T2), and type Ill changes are due to sclerosis (low on T1 and T2).

How well did you know this?
1
Not at all
2
3
4
5
Perfectly
40
Q

(MSK) 39 Regarding giant cell tumours, which of the following statements is true?

(a) GCT usually regresses during pregnancy

(b) It is a highly malignant lesion

(c) Surgical resection is usually curative

(d) The majority arise in the spine

(e) Vertebral body involvement is more common than the posterior elements

A

(e) Vertebral body involvement is more common than the posterior elements

GCTs are usually benign lesions.

Malignancy occurs in 5-10°/o of cases and is usually secondary to previous radiation therapy.

The majority of spinal lesions arise within the sacrum.

Vertebral involvement accounts for only 7% of cases: thoracic spine is the most common location, followed by cervical and lumbar regions.

They typically increase in size during pregnancy, thought to be due to hormonal influences.

GCTs tend to be locally aggressive and complete surgical resection is uncommon; adjuvant radiotherapy is often administered.

Recurrence occurs in 40- 60%, and is suggested on plain film by the presence of new areas of osseous destruction.

How well did you know this?
1
Not at all
2
3
4
5
Perfectly
41
Q

(MSK) 2 A gentleman presents to A&E after trauma. Plain radiographs of the cervical spine are taken. There is an abrupt transition in the alignment of the cervical spine at CS-6, with anterolisthesis of CS on CG by 3/4 of a vertebral body’s width.
Which of the following is incorrect?

(a) There is a high incidence of cord injury

(b) This is a stable dislocation

(c) The posterior ligament complex is disrupted

(d) The anterior longitudinal ligament is disrupted

(e) The facets may be in a ‘batwing’ or ‘bow-tie’ configuration

A

(b) This is a stable dislocation

Given the extent of anterolisthesis (>50% of a vertebral body), this is most likely to represent bilateral, rather than unilateral facet dislocation, and therefore an unstable injury.

How well did you know this?
1
Not at all
2
3
4
5
Perfectly
42
Q

2 A middle aged man with progressive lower-extremity weakness undergoes an MRI examination of the spine. T2W images demonstrate multiple small vessel flow voids on the cord pial surface, but no flow voids within the cord. The cord is enlarged, with oedema sparing the periphery. Which type of spinal cord arteriovenous malformation does this represent?

(a) Type I

(b) Type II

(c) Type Ill

(d) Type IV

(e) Type V

A

(a) Type I

This is a spinal dural AV fistula (type I), the fistula is present within the dura, producing distended draining veins on the pial surface and cord oedema secondary to venous hypertension.

How well did you know this?
1
Not at all
2
3
4
5
Perfectly
43
Q

5 An MRI examination of the lumbar spine demonstrates 5 mm of intravertebral disc tissue protruding beyond the margin of the vertebral body over 60% of the vertebral body circumference. What is the most appropriate diagnosis?

(a) Annular disc bulge

(b) Broad based disc herniation

(c) Focal disc herniation

(d) lntravertebral herniation

(e) Disc sequestration

A

(a) Annular disc bulge

In an annular disc bulge, disc tissue extends beyond the adjacent vertebral bodies by at least 3mm for more than 50% of the disc circumference. This is associated with degenerative disease and is not regarded as a true herniation.

How well did you know this?
1
Not at all
2
3
4
5
Perfectly
44
Q

34 Which of the following is a contraindication to percutaneous vertebroplasty?

(a) Myeloma metastasis

(b) Vertebral body haemangioma

(c) Previous vertebroplasty in an adjacent vertebra

(d) Fractures involving the posterior elements

(e) Sclerotic metastases

A

(d) Fractures involving the posterior elements

Other absolute contraindications include bleeding diathesis, acute fractures not responding to bisphosphonate treatment within 2 weeks and those where the level of collapse cannot be clearly defined.

How well did you know this?
1
Not at all
2
3
4
5
Perfectly
44
Q

(MSK) 16 A middle aged man presents with low back pain and. Faecal incontinence. MR imaging of the lumbar spine is performed and a diagnosis of chordoma is subsequently made. Regarding the chordomas, which of the following is incorrect?

(a) They typically have poor uptake of 99mTc-MDP

(b) They usually cause extensive local bone destruction

(c) They most frequently arise in the sacrum or coccyx

(d) They may have a narrow zone of transition

(e) Metastasis is common

A

(e) Metastasis is common

Chordomas arise from notochord remains and are therefore limited to the clivus, spine, sacrum and coccyx. Metastasis is uncommon, but when it does occur, lung secondaries are typical. Tumour size (the average size of a sacrococcygeal chordoma is 10 cm), lytic nature and location are important clues to the diagnosis.

How well did you know this?
1
Not at all
2
3
4
5
Perfectly
45
Q

(MSK) 36 A 35 year old man involved in an RTA presents to A&E with lower neck pain. The mechanism of injury is thought be one of flexion. Cervical and thoracic spine films are obtained. Which of the following flexion fractures would you describe as being unstable?

(a) Anterior subluxation

(b) Clay-Shoveler’s fracture

(c) Flexion teardrop fracture

(d) Unilateral facet joint dislocation

(e) Wedge compression fracture

A

(c) Flexion teardrop fracture

Neutral films infer stability based on fracture type; stability is a function of ligamentous injury and thus cannot be implied with 100% accuracy, if doubt remains MRI or flexion/ extension views should be obtained. The other type of unstable flexion injury is a bilateral facet joint dislocation. Unstable extension injuries include Hangman’s fracture and hyperextension-dislocation fracture; stable extension injuries include posterior arch of C1 fracture, laminar fracture, Pillar fracture, and extension teardrop fracture. Jefferson’s fracture is an unstable compression fracture, burst fracture a stable one. ‘Complex’ unstable fractures include odontoid fracture and atlanto-axial disassociation.

How well did you know this?
1
Not at all
2
3
4
5
Perfectly
46
Q

(Ped) 39 A 6 year old child is a passenger in an RTA. The cervical spine cannot be clinically cleared and C-spine films are requested. Which of the following findings are trauma-related rather than a normal variant?

(a) Anterior subluxation of C2 on C3

(b) Anterior wedging of the C3 vertebral body

(c) Atlanta-dens interval of 5 mm

(d) Lucent line in C3 spinous process

(e) Prevertebral space of 8 mm anterior to C3

A

(d) Lucent line in C3 spinous process

Cervical spine injuries in children are usually located between the occiput and C2-C3, because the spine is hypermobile (ligament laxity), there is incomplete ossification of the odontoid process, a relatively large head, and weak neck muscles. The other options are normal variants, others include absent cervical lordosis, intervertebral widening, and pseudospread of the atlas on the ‘Peg’ view (pseudo-Jefferson fracture). Pseudo-widening of the prevertebral soft tissues can be normal, related to expiration.

How well did you know this?
1
Not at all
2
3
4
5
Perfectly
47
Q

64 A man with lower back pain has a plain lumbar radiqgraph. The LS vertebral body has slipped forward on S1 by 60% of the body diameter. What grade spondylolisthesis does this represent?

(a) I

(b) II

(c) III

(d) IV

(e) V

A

(c) III

Spondylolisthesis is graded from I-IV with each grade corresponding to 25% of displacement.

How well did you know this?
1
Not at all
2
3
4
5
Perfectly
48
Q

(MSK) 3. A 56 year old motorcyclist has a trauma series of plain films following a road traffic accident. On evaluation of the lateral cervical spine film, which of the following soft tissue parameters would be a concerning feature?

a. Predental space of 3mm

b. Nasopharyngeal space of 7mm

c. Retropharyngeal space of 10mm

d. Retrotracheal space of 20mm

e. Decreased disc space at the C5/6 level

A
  1. c. Retropharyngeal space of 10mm

This is too wide for the retropharyngeal space. The correct acceptable limits for soft-tissue measurements are as follows:
_ Predental space 3mm in adults, 5mm in children.
_ Nasopharyngeal space (anterior to C1) 10 mm.
_ Retropharyngeal space (C2–C4) 5–7 mm.
_ Retrotracheal space (C5–C7) 22 mm.
Disc spaces should be roughly equal throughout the cervical spine. Narrowing of a disc space is due to degenerative change, but widening would be a more concerning feature.

How well did you know this?
1
Not at all
2
3
4
5
Perfectly
49
Q

@# (Ped) 5. A ten year old boy presents with a history of progressive gait abnormalities. Plain radiographs of the thoraco-lumbar spine show widening of the spinal canal at T8-L1. MRI demonstrates an eccentric, ill-defined, homogeneous intramedullary lesion which is hypointense to the cord on T1 and hyperintense on T2. There is patchy, irregular enhancement post-contrast. What is the most likely diagnosis?

a. Lipoma

b. Ependymoma

c. Astrocytoma

d. Ganglioglioma

e. Haemangioblastoma

A
  1. c. Astrocytoma

Astrocytoma of the spinal cord is the most common intramedullary neoplasm in children. They most commonly occur in the thoracic region (thoracic 67%, cervical 49%, conus medullaris 3%). The most common presentation is with pain and sensory deficit but they can also present with motor and gait abnormalities. Plain radiographs may demonstrate scoliosis, bone erosion and widened interpedicular distance. On MRI, the lesion is usually seen as an eccentric, homogeneous, extensive, ill-defined cord tumour that is iso- or hypointense to the cord on T1 and hyperintense in T2. There is patchy irregular gadolinium enhancement. Tumour cysts and syrinx are also common. Patients with low-grade astrocytomas have a 95% five-year survival. It is often difficult to differentiate an astrocytoma from ependymoma of the spinal cord on imaging. In this case, the age of the patient, tumour location, tumour irregularity and eccentric position within the medullar favour astrocytoma.

How well did you know this?
1
Not at all
2
3
4
5
Perfectly
50
Q

(MSK) 13. A middle-aged woman undergoes an MRI of the lumbar spine for longstanding lower back pain. She has no specific neurological signs and is otherwise well. MRI shows some lower lumbar spine facet joint arthropathy and a 2_2 cm well-defined rounded lesion in the L3 vertebral body. This displays high signal on both the T1 and T2 sequences. The most likely explanation for this lesion is:

a. Discitis

b. Lymphoma

c. Myeloma

d. Metastatic deposit

e. Haemangioma

A
  1. e. Haemangioma

This is most likely to be a benign haemangioma. These are relatively common lesions seen as incidental findings on spinal imaging. High signal on T1 imaging is indicative of the presence of fat within the lesion. All the other conditions would give a low-signal lesion on T1 imaging.

How well did you know this?
1
Not at all
2
3
4
5
Perfectly
51
Q

(MSK) 18. A 23 year old man sustains a Jefferson fracture to his cervical spine following an injury in which he dived into a shallow swimming pool, hitting his head on the bottom. Which of the following regarding his injury is incorrect?

a. Displacement of the lateral masses of C1 relative to the dens on an odontoid view indicates a transverse ligament rupture

b. Associated C2 fracture will be present in up to 30% of cases

c. Jefferson fractures are usually associated with a neurological deficit

d. Up to 50% are associated with a further cervical spine injury

e. There may be associated vertebral artery injury

A
  1. c. Jefferson fractures are usually associated with a neurological deficit

Jefferson fractures are not usually associated with neurological deficit. Although there may be retropulsion of fragments into the vertebral canal, spinal cord injury is rare due to the large dimensions of the canal at this level. Vertebral artery injury, however, must be considered and if there is concern either CTA or MRA imaging should be considered.

How well did you know this?
1
Not at all
2
3
4
5
Perfectly
52
Q

(MSK) 27. A 52 year old woman presents to her GP with a longstanding history of lower back pain which has suddenly worsened in severity over the past few days. An urgent MRI scan of the lumbar spine shows a right paracentral disc protrusion at the L4/L5 level. The disc impinges on the lateral recess at this level. The most likely nerve to be affected is the:

a. Cauda equina

b. Lumbar plexus

c. Right L4

d. Right L5

e. Right S1

A
  1. d. Right L5

The right L5 nerve root is the most likely to be affected as it will be sitting in the right lateral recess at the L4/5 level. The L4 nerve root will be at the exit foramen and therefore if the protrusion affects only the lateral recess then this nerve will already have exited and therefore not be affected.

How well did you know this?
1
Not at all
2
3
4
5
Perfectly
53
Q

(MSK) 34. A 25 year old male is involved in a 60 mph road traffic accident with a head-on collision. He was wearing a seat-belt but his car did not have an air-bag. A screening lateral radiograph of the cervical spine shows the following findings: an angular kyphosis centred at C4/C5, a 1mm anterior slip of C4 on C5, and widening of the interspinous space posteriorly. What is the likely mechanism for this injury?

a. Lateral compression

b. Flexion

c. Extension

d. Combination

e. Rotation

A
  1. b. Flexion

This describes the typical appearance for a flexion injury as well as the typical mechanism. This would represent a potentially unstable fracture and immobilisation would be essential until further management decisions are made. Flexion teardrop injuries are more common in the lower cervical spine and extension teardrop injuries are more common in the upper cervical spine

54
Q
  1. A 15 year old girl presents with suspected optic neuritis. Post-contrast T1-weighted imaging shows enhancement and mild enlargement of the right optic nerve. Several months later the girl re-presents with symptoms of myelitis. Sagittal T2-weighted imaging of the spine demonstrates intramedullary T2 hyperintensity and cord expansion extending from C1 to T1. At the time of the initial and subsequent presentations, brain imaging was normal. What is the most likely diagnosis?

a. Multiple sclerosis

b. Sarcoidosis

c. Devic syndrome

d. Neurofibromatosis

e. Ependymoma

A
  1. c. Devic syndrome (DS)

Devic syndrome (neuromyelitis optica (NMO)) is a severe demyelinating syndrome characterized by optic neuritis and acute myelitis. There is substantial evidence that DS is distinct from multiple sclerosis (MS). DS may spare the brain, the spinal lesions are much larger than in MS and the serum autoantibody NMO-IgG is greater than 90% specific for DS patients and is not detected in classical MS patients.
The proposed diagnostic criteria for Devic syndrome are optic neuritis and acute myelitis plus at least two of three supporting criteria:
1. Contiguous S.cord MRI lesion extending over > or equal to three vertebral segments.
2. Brain MRI not meeting diagnostic criteria for multiple sclerosis.
3. NMO-IgG seropositive status.

55
Q

(MSK) 47. A 24 year old woman presents with worsening frontal headaches and a sixth nerve palsy. A non-enhanced CT shows a lesion situated within the clivus with associated bony destruction; there is soft-tissue extension into the nasopharynx. MRI shows a large inter-osseous mass which is isointense to brain T1-weighted imaging and hyperintense on T2. The most likely diagnosis is:

a. Sphenoid sinus cyst

b. Meningioma

c. Nasopharyngeal carcinoma

d. Metastasis

e. Spheno-occipital chordoma

A
  1. e. Spheno-occipital chordoma

The most likely cause is a spheno-occipital chordoma. This is associated with bony destruction in 90% of cases and is most usually within the clivus. Other sites include the sella, petrous temporal bone, floor of middle cranial fossa and jugular fossa. Sacrococcygeal chordoma is the most common subtype of chordoma and is usually located with the fourth or fifth sacral segments. Vertebral/spinal chordoma accounts for only 15–20% of all chordomas & is most often situated in cervical spine.

56
Q

(Ped) 50. A seven year old boy attends A&E following a road traffic accident in which he hit the kerb and came off his bike. He immediately complained of neck pain and was immobilised at the scene. Initial plain radiographs including a swimmer’s view fail to demonstrate the entire cervical spine adequately. There remains clinical concern regarding his cervical spine. The next step should be:

a. Trauma oblique view

b. Flexion/extension radiographs

c. MRI C spine

d. CT C spine

e. Repeat swimmer’s view with traction on arms

A
  1. c. MRI C spine

MRI should be the next investigation in a child of this age group. A ligamentous injury is more likely and therefore CT of the C spine may be falsely reassuring. However, in some centres the availability of MRI may be limited and a CT will often be performed in the first instance. Flexion/extension views should not be done in the acute setting. A trauma oblique view is performed in some institutions instead of a swimmer’s view & may be used to clear the cervicothoracic junction in particular.

57
Q

(MSK) 51. An A&E SHO has asked you to review a paediatric cervical spine plain film which has been performed on a child who has been involved in a road traffic accident. He is unsure as to whether or not the appearances are normal for a paediatric cervical spine film. Which of the following findings is more likely to represent a true cervical spine injury than a normal variant?

a. Absence of usual cervical lordosis

b. Widening of the prevertebral soft tissues in expiration

c. Increased distance between the tips of the C1 and C2 spinous processes in flexion

d. Wedging of the anterior aspect of the C3 vertebral body

e. A 7mm gap between the occipital condyles and the condylar surface of the atlas

A
  1. e. A 7mm gap between the occipital condyles and the condylar surface of the atlas

This is highly suggestive of craniocervical injury; these injuries are often fatal and are often caused by sudden deceleration. Radiologic evaluation of this injury can be difficult but is crucial in determining further management. The remainder of the findings above can all be normal variants in the paediatric cervical spine and therefore should be interpreted with care.

58
Q
  1. A male patient presents with severe back pain following surgery seven months previously. MRI shows extensive clumping of roots of the cauda equina to the right of the midline, suggesting an intrathecal mass. There is no enhancement post-gadolinium administration. What is the most likely diagnosis?

a. Intrathecal metastases

b. Nerve root sheath tumour

c. Arachnoiditis

d. Extradural haematoma

e. Prolapsed intervertebral disc

A
  1. c. Arachnoiditis

Arachnoiditis is an intrathecal inflammatory reaction which usually results from iatrogenic injury. Previously, it was associated with the injection of contrast agents during myelography. Now, it is most commonly seen following lumbar surgery. Intradural infections (TB, fungal, parasitic) are rare causes. Radiologically, arachnoiditis appears as nerve root adhesion. Commonly the nerves are clumped together simulating a solid mass, although the nerves may also adhere to the dura, giving the appearance of an ‘empty thecal sac’. In severe cases, the thecal sac may become loculated and compartmentalised. Clumped nerve roots suggest arachnoiditis and the lack of enhancement makes tumours unlikely.

59
Q
  1. A 60 year old man presents with back pain and progressive paraparesis. T1-weighted MR imaging shows loss of T9–T10 disc space with hypointense signal involving multiple contiguous vertebral bodies. Skip involvement of T3 and L1 vertebral bodies is evident. T2-weighted imaging shows a large paraspinal and prevertebral mass. An anterior epidural collection is seen compressing the cord. On post-contrast T1-weighted imaging the vertebral bodies show inhomogeneous enhancement and the paraspinal mass shows peripheral enhancement with central necrosis. What is the most likely cause?

a. Pyogenic infection

b. Lymphoma

c. Sarcoidosis

d. Tuberculosis

e. Metastases

A
  1. d. Tuberculosis

The vertebral column is the most common site of osseous involvement of tuberculosis (TB), usually as the result of haematogenous spread from pulmonary involvement. The lower thoracic and upper lumbar vertebrae are the most frequently affected. It can be difficult to differentiate between pyogenic and TB spondylodiscitis on imaging but several features favour a diagnosis of TB: multilevel involvement/skip lesions, relative sparing of the disc spaces, large pre/paravertebral collections which are more likely to calcify, subligamentous spread and meningeal involvement

60
Q

(MSK) QUESTION 7
A 19-year-old man, the unrestrained driver in a high-energy road traffic accident, has been brought by ambulance to the Emergency Department. A lateral cervical spine radiograph shows an anterior wedge fracture of C5 with a retropulsed bony fragment. What was the likely predominant force acting on the cervical spine at the time of injury?

A Compression

B Distraction

c Extension

D Flexion

E Shearing

A

A Compression

61
Q

QUESTION 17
A 77-year-old gentleman suffers a ruptured abdominal aortic aneurysm for which he undergoes emergency surgery. Early in the postoperative period he develops acute lower back pain and is incontinent. An urgent MRI is performed. What are the most likely radiological findings?

A Enhancing, heterogeneous intramedullary mass lesion

B Extradural collection with peripheral enhancement

C Focal high T2 signal within the spinal cord with mild cord swelling

D Large central disc herniation at the affected level with associated high T2 signal within the cord

E Marked diffuse swelling of the spinal cord at the affected level

A

C Focal high T2 signal within the spinal cord with mild cord swelling

These are the expected findings in spinal cord infarction and this clinical history is typical

62
Q

(Ped) QUESTION 17
A 5-year-old boy is involved in a traffic accident and is complaining of neck pain. Which of the following statements is true regarding the cervical spine radiograph?

A Subluxation of up to 7 mm of C2 anteriorly on C3 is normal.

B Subluxation of up to 3 mm of C2 posteriorly on C3 is normal.

C The distance between the anterior arch of Cl and the dens can be up to 5 mm.

D The soft tissues anterior to C2 must be no wider than 1/4 of the width of the C2 vertebral body.

E The soft tissues anterior to C6 must be no wider than 1/2 of the width of the C6 vertebral body

A

C The distance between the anterior arch of Cl and the dens can be up to 5 mm.

Children may also get spinal cord injury without radiographic abnormality (SCIWORA) as the ligaments are lax; therefore, MRI may be necessary.

63
Q

QUESTION 29
A 35-year-old man is admitted from outpatient clinic with a history of worsening lower back pain and leg weakness. Plain lumbar radiographs reveal expansion of the spinal canal at L3—4 level. He has an urgent MRI which shows a lobulated extramedullary mass at this level, causing nerve root compression. The mass is hyperintense to the spinal cord on T2w images and there is an associated paravertebral mass. What is the most likely diagnosis?

A Astrocytoma

B Disc extrusion

C Ependymoma

D Meningioma

E Neurinoma

A

E Neurinoma

Neurinomas and meningiomas are the commonest intradural extramedullary tumours. Neurinomas may occur at any level whereas meningiomas tend to be thoracic and are very rare in the lumbar spine.

64
Q

QUESTION 32
A 67-year-old woman with known osteoarthritis presents with lower back pain radiating down her left leg. She has an MRI of the lumbar spine which shows a lesion at the L4—5 facet joint with compression of the thecal sac at this level. The lesion is of intermediate signal on T2w images and is displacing the ligamentum flavum. What is the most likely diagnosis?

A Astrocytoma

B Disc protrusion

C Ependymoma

D Osteophyte

E Synovial cyst

A

E Synovial cyst

Synovial cysts may be solid with cartilaginous or myxomatous components

65
Q

QUESTION 36
A 12-year-old girl presents with a 3-week history of lower back pain, malaise and a low grade fever. A lateral lumbar spine radiograph shows features suspicious of discitis. Which one of the following findings would support tuberculous rather than pyogenic infection?

A Bony bridging of affected vertebrae

B Destruction of the adjacent endplates

C Large paravertebral abscess

D Marked bony sclerosis

E Preserved vertebral body height

A

C Large paravertebral abscess

It is not possible to reliably differentiate between pyogenic and tuberculous spondylitis but some features may be helpful. Pyogenic infection tends to be rapidly progressive, whereas tuberculous infection is more insidious. In pyogenic infection there is a marked sclerotic response; bridging of affected vertebrae is an early sign but vertebral collapse and paravertebral abscesses are not prominent features. In tuberculous infection a large paravertebral abscess (which may contain calcification) is a more common finding and there is typically marked vertebral body collapse but less bony sclerosis.

66
Q

(MSK) QUESTION 37
A 20-year-old man is brought to the Emergency Department after diving into the shallow end of a swimming pool at a party. Witnesses describe a hyperflexion injury to the cervical spine. A fracture is identified on the lateral cervical radiograph. What is the most likely fracture configuration given the mechanism of injury?

A Anterior wedge fracture

B Burst fracture, with an anterior wedge fracture and a retropulsed fragmeht

C Fracture dislocation, with anterior dislocation of the more cranial vertebra and an avulsion fracture of the superoanterior margin of the more inferior vertebra

D Fracture of the pars interarticularis, in association with an avulsion fracture through the anteroinferior margin of the vertebra above

E Posterior clements fracture with anterior vertebral wedging

A

A Anterior wedge fracture

67
Q

(MSK) QUESTION 45
A radiograph of the thoracolumbar spine in a 75-year-old woman with a history of back pain shows an anterior wedge compression fracture of Tl 1. What influence does this observation have on the likelihood of a subsequent fracture?

A Doubles the risk of another vertebral fracture

B Doubles the risk of a proximal femoral fracture

C Has little effect on the risk of a subsequent fragility fracture

D Triples the risk of another vertebral fracture

E Triples the risk of a proximal femoral fracture

A

B Doubles the risk of a proximal femoral fracture

Vertebral fractures are not only the most common fragility fracture, but also serve as powerful predictors of subsequent fracture. A vertebral fracture increases the likelihood of a further vertebral fracture fivefold, and doubles the risk of subsequent femoral neck fracture.

68
Q

@# (MSK) QUESTION 56
A 40-year-old builder is admitted unconscious to the Emergency Department following an accident at work. Derails of the accident are unclear, but one witness describes scaffolding collapsing. He undergoes an emergency CT head and cervical spine, which reveals lateral displacement of both the lateral masses of Cl. How may such an injury be described?

A Atlanto-axial subluxation

B Clay shoveller’s fracture

C Dens fracture

D Hangman’s fracture

E Jefferson’s fracture

A

E Jefferson’s fracture

69
Q

QUESTION 59
A 3-year-old child with scoliosis attends for an MRI of the spine. The sagittal images show apparent thinning of the lower thoracic spinal cord and abnormal signal within the cord. Axial images show that the cord returns normal signal and is split in two over the length of two vertebral bodies, with a cleft of CSF between the two hemicords. Which of the following may be associated with these findings?

A Atrophy of the filum terminale

B Bifid vertebrae

C Descent of the cerebellar tonsils below the foramen magnum

D High conus medullaris

E Narrowing of the interpedicular distance

A

B Bifid vertebrae

This describes diastematomyelia which has several associations, including hemivertebrae and bifid vertebrae.

70
Q

(MSK) QUESTION 60
A 35-year-old man is brought to the Emergency Department following a fallfrom a motorcycle. The lateral cervical radiograph shows a well-marginated triangular area of bone at the anterosuperior margin of C5. The cortical margin of the adjacent vertebral body is smooth. The rest of the spine is normal. What is the most likely diagnosis?

A Avulsion of the anterior longitudinal ligament

B Limbus vertebra

C Schmorl’s node

D Teardrop fracture of C5

E Unfused ring epiphysis

A

B Limbus vertebra

71
Q

QUESTION 61
A 7-year-old boy with deafness and a low hairline has cervical spine radiographs to investigate limited neck movements. These show fusion of the vertebral bodies and posterior elements of C2, C3 and C4. On review of his previous imaging, there is a left shoulder radiograph which shows elevation and rotation of the scapula. Which of the following is the most likely diagnosis?

A Klippel-Feil syndrome

B Madelung deformity

C Morquio disease

D Swyer-James syndrome

E Turner’s syndrome

A

A Klippel-Feil syndrome

Sprengel deformity of the scapula is a recognised association of Klippel-Feil syndrome.

72
Q

QUESTION 75
A 72-year-old man presents with increasing leg weakness and incontinence. There is a past medical history of previous TB of the spine. An MRI of the lumbar spine shows a tapered appearance of the lower end of the subarachnoid space and the thecal sac appears empty but thick-walled. Which one of the following is the most likely diagnosis?

A Arachnoiditis

B Discitis

C Dural arteriovenous malformation

D Epidural abscess

E Spinal sarcoidosis

A

A Arachnoiditis

Arachnoiditis may be iatrogenic or may occur due to intradural infections, trauma, spinal subarachnoid haemorrhage and rarely intraspinal tumours.

73
Q

(MSK) QUESTION 76
A 35-year-old woman with back pain has radiographs taken of her lumbosacral spine. The frontal radiograph reveals narrowing of the right sacroiliac joint with significant periarticular sclerosis. The contralateral sacroiliac joint is normal. The lumbar spine is within normal limits for age. What is the most likely diagnosis?

A Ankylosing spondylitis

B Infection

C Osreitis condensans ilii

D Psoriatic arthropathy

E Reiter’s syndrome

A

B Infection

Infection is the commonest cause of unilateral sacroiliitis, and TB should be considered as a possible organism. Ankylosing spondylitis and osteitis condensans ilii have symmetrical appearances, whilst Reiter’s syndrome is bilateral (but asymmetric). Psoriatic arthropathy produces bilateral disease, which is symmetrical in most cases.

74
Q

(MSK) 4. A 63-year-old female is being worked up for a left total hip replacement. She has a history of RA. As part of the routine pre-operative assessment in your hospital a cervical spine radiograph is requested. This demonstrates that there is widening of the pre-dental space, with the anterior arch of C1 located anterior to the lower part of the body of C2. The dens is not clearly visible. This appearance is constant on the flexion view. The patient is asymptomatic. What do you think these findings represent?

A. Degenerative change.

B. Pannus erosion of dens.

C. Atlanto-axial subluxation.

D. Erosion of the occipital condyles.

E. Atlanto-axial impaction.

A
  1. E. Atlanto-axial impaction.

This is a more severe form of atlanto-axial subluxation where the C1-2 facets collapse and there is invagination of the dens of C2 into the foramen magnum. As such, the dens is not visible on the lateral radiograph. The key feature, apart from widening of the pre-dental space (which can also be caused by pannus eroding the dens or more commonly atlanto-axial subluxation), is that the anterior arch of C1 lies in front of the lower portion of C2, whereas it normally lies anterior to the dens.

75
Q
  1. A 45-year-old man presents with a 4-month history of worsening lowerback pain radiating into the right lower extremity with weakness. An MRIscan of lumbar spine shows a 3-cm well-defined ovoid lesion eccentricallyplaced at the conus medullaris, the location of which is felt to be intraduraland extramedullary. It is hypointense on T1WI and hyperintense on T2WI.There are flow voids and haemorrrhage within the lesion. It enhances avidlypost injection of contrast. Which of the following lesions fits best with the imaging findings?

A. Meningioma.

B. Schwannoma.

C. Neurofibroma.

D. Paraganglioma.

E. Metastasis.

A
  1. D. Paraganglioma.

Paragangliomas are rare intradural extramedullary tumours that are usually benign and haveimaging characteristics as those described. Schwannomas, neurofibromas, and meningiomas aremuch more common, but do not typically contain vascular flow voids or areas of haemorrhage. Meningiomas may have a dural tail or foci of calcification. Schwannomas and neurofibromascan be difficult to distinguish on imaging. They are typically isointense on T1WI and markedlyhyperintense on T2WI. Enhancement may be intense and homogenous or peripheral.Neurofibromas tend to encase, rather than displace, nerve roots.Leptomeningeal metastases present with three different imaging patterns:
1) Diffuse, thin enhancing coating of the surface of the spinal cord and nerve roots.
2) Multiple small enhancing nodules on the surface of the cord and nerve roots.
3) A single mass in the lowest part of the thecal sac.

76
Q

(MSK) 1. A 50-year-old man who has been previously well presents with low back pain. Plain film reveals an osteolytic midline lesion in the lower sacrum containing secondary bone sclerosis in the periphery, as well as amorphous peripheral calcifications. A lateral film shows anterior displacement of the bladder and rectum. He subsequently develops faecal incontinence. No additional lesions were discovered after imaging of the whole spine. What is the most likely diagnosis?

A. Osteomyelitis.

B. Ewing’s sarcoma.

C. Chordoma.

D. Myeloma.

E. Sacrococcygeal teratoma

A
  1. C. Chordoma.

Plain film is very insensitive for detecting sacral lesions. Metastatic disease is much more common in the sacrum than primary malignancy.

Chordoma is the most common primary sacral lesion. It is derived from the embryonic remnants of the notochord and is thus almost always found in the midline or a paramedian location with respect to the spine.

It is most commonly found in the sacrum (50%), clivus (35%), and vertebrae (15%).

A chordoma manifests as a destructive, lytic lesion, commonly with internal calcifications, at both plain radiography and CT. A large presacral soft-tissue component is usually present, as are soft-tissue components within the sacrum and sacral canal.

Symptoms can include pain, sciatica, and rectal bleeding as well as other bowel and bladder symptoms, reflecting compromise of sacral nerves.

The tumours can extend across the adjacent disc space and sacroiliac joint.

On MRI, chordomas demonstrate low to intermediate signal intensity on T1WI and prominent increased signal intensity on T2WI. Enhancement of the soft-tissue components is variable, yet often moderate, on both CT and MR images.

Chordomas demonstrate a prominent vascular stain at angiography.

They are locally aggressive and develop in locations that do not permit easy surgical cure. There is an almost 100% recurrence rate; tumour seeding along biopsy tracts and surgical incisions can lead to multicentric local recurrences.

Metastasis occurs in 5–43% to liver, lung, regional lymph nodes, peritoneum, skin, and heart.

The 5-year survival rate is 66% for adults.

Osteomyelitis in the sacrum is most often due to contiguous spread from a suppurative focus and we are told this patient was previously well.

Ewing sarcoma would occur at a younger age (peaking at 15 years, 90% manifest between the ages of 5 and 30).

In the case of myeloma it would be atypical for the rest of the spine to be uninvolved.

The sacrococcygeal region is the most common location of teratomas discovered in infancy. It is only rarely discovered in adulthood. Teratomas are composed of a mixture of cystic and solid components.

The other lesions to be included in the differential diagnosis of such a sacral mass are metastasis, sarcomas, GCT, chondrosarcoma, and ependymoma.

77
Q
  1. A 45-year-old woman presents with a several month history of neck painand gradually progressive weakness and paraesthesia in the upper limbs. AnMRI scan of the cervical spine is performed and this shows a well-definedcentral intramedullary mass in the mid-cervical spinal cord. The massis generally slightly hyperintense on T2WI, but also has a few low signalperipheral areas. It enhances homogeneously with gadolinium. What is themost likely diagnosis?

A. Astrocytoma.

B. Metastasis.

C. Haemangioblastoma.

D. Ganglioglioma.

E. Ependymoma.

A
  1. E. Ependymoma.

Ependymoma is the most common intramedullary neoplasm in adults. It tends to be centrally located within the cord, unlike astrocytoma, which can be eccentric.

Astrocytoma can have alonger segment of cord involvement than ependymoma and may have a more infiltrative margin.

The peripheral low signal areas seen on T2WI in ependymoma are related to haemosiderin deposition from prior haemorrhage.

Haemangioblastoma is more often seen in the dorsal cord than the cervical cord and is typically a small well-defined lesion. It may have an associated cord cyst or syrinx. Flow voids may be seenwithin the lesion, from dilated vascular channels.

Ganglioglioma is a very rare, slow-growing tumour of low malignant potential. The imaging appearance is non-specific, but there are some findings that may suggest the diagnosis. Compared with other spinal cord tumors, gangliogliomas are more likely to involve long segments of thecord (greater than four levels, up to the whole cord), to be associated with bone erosion orscalloping, to have tumoral cysts, and to have areas of mixed high signal on precontrast T1WI.

Intramedullary metastasis represents less than 5% of intramedullary lesions. They usually occur inthe setting of advanced malignant disease, typically from a lung or breast primary. The spinal cord oedema can seem out of keeping with the small size of the metastatic lesion.

78
Q

@# (Ped) 27. A 6-year-old child attends A&E with neck pain and tenderness after landing badly whilst trampolining in a neighbour’s back garden. Which of the following findings is most concerning?

A. C2/3 subluxation.

B. Overhang of the lateral masses of C1 on C2 of 6 mm.

C. An atlanto-dens interval (ADI) of 6 mm.

D. Prevertebral soft tissue of 6 mm at C3.

E. Anterior wedging of C3.

A
  1. C. An atlanto-dens interval (ADI) of 6 mm.

There are appearances that are normal for a paediatric cervical spine, which would be considered pathological in an adult and of which it is important to be aware.

Pseudo-subluxation at C2/3 and C3/4 is common (it was observed in 46% of patients less than 8 years old at C2/3 in one study and has been seen up to 14 years).

The anterior aspects of the spinous processes of C1, C2, and C3 should line up within 1 mm of each other on flexion and extension (assessed by drawing the posterior cervical line, a line from the anterior aspect of the spinous process of C1 to the equivalent point at C3).

The anterior aspect of the spinous process of C2 is allowed to pass through, touch, or lie up to 1 mm behind the posterior cervical line in physiological subluxation.

The normal ADI (the distance between the anterior aspect of the dens and the posterior aspect of the ring of the atlas) in adults is 3 mm, but is normal up to 5 mm in children.

‘Pseudo-Jefferson’ fractures (pseudospread of the lateral masses of C1 on C2) can be seen on the peg view, and up to 6 mm of displacement is common up to 4 years old and may be seen in patients up to 7 years old.

Anterior wedging of up to 3 mm of the vertebral bodies should not be confused with compression fracture. This finding can be profound at C3.

Prevertebral swelling of 6 mm or less is considered normal at C3. Widening of prevertebral soft tissues in children can be due to expiration, and if suspected, a repeat lateral film in inspiration and mild extension should be performed.

79
Q

@# 48. A 22-year-old woman presents with upper and lower limb neurological symptoms and signs. She is subsequently discovered on MRI to have a mass in the cervical spinal cord. Which of the following features on MRI is goingto point more towards a diagnosis of spinal cord astrocytoma, rather than ependymoma?

A. Predominant T2WI high signal.

B. Homogeneous enhancement post gadolinium.

C. Short segment of cord involvement.

D. Eccentrically placed lesion in the cord.

E. Sharply marginated lesion.

A
  1. D. Eccentrically placed lesion in the cord.

Table 6.2 illustrates the diagnostic features ofastrocytoma and ependymoma

80
Q

@# 49. A 52-year-old woman presents with gradually increasing gait disturbance and lower limb sensory symptoms. An MRI of her spine is performed and this shows an anteriorly placed intradural, but extramedullary spinal mass.It is fairly markedly low signal on T1WI and T2WI, and shows only miminal patchy enhancement post administration of intravenous gadolinium. What isthe most likely diagnosis?

A. Neurofibroma.

B. Schwannoma.

C. Lymphoma.

D. Metastasis.

E. Meningioma.

A
  1. E. Meningioma.

Spinal meningiomas are typically iso- to hypointense on T1WI and slightly hyperintense onT2WI. There is usually strong and homogeneous enhancement with gadolinium. However, some meningiomas may contain calcification and are typically the only intradural extramedullarytumours to do so.

Some meningiomas can be heavily calcified and such a meningioma is being described in the question. These will remain dark on all MRI sequences and demonstrate onlylittle contrast uptake (in the non-calcified areas).

Schwannomas, neurofibromas, and metastases would not typically be hypointense on T2WI.

Meningeal lymphomas are very rare and usually manifest as diffuse thickening of nerve roots and/or multiple enhancing nodules.

81
Q

@# 50. You are asked to protocol an MRI scan that is specifically being performed to look for vertebral metastatic disease. The radiographer complains that you have asked for too many sequences. Which of the following sagittal sequences is likely to be least helpful for the purposes of your examination?

A. STIR.

B. T2 fast SE with fat saturation.

C. T2 fast SE.

D. T1 fast SE.

E. T1 GE out of phase.

A
  1. C. T2 fast SE.

T2 fast SE is probably the least useful sequence when specifically looking for vertebral marrow deposits because the metastases are less conspicuous, typically being high signal on a background of high-signal fatty marrow.

On STIR and T2 fast SE with fat saturation, the metastases typically stand out as being of increased signal on a background of dark marrow because of the fat saturation techniques.

On T1 fast SE sequences, the metastases typically stand out as being lowsignal on a background of high-signal fatty marrow.

Finally, T1 GE out-of-phase imaging is also good for looking for vertebral metastatic disease. This is a sequence with a specific echo time corresponding to the time it takes for water and fat protons to move exactly 180° out of phase. In the normal adult human, the medullary bone of the vertebral bodies contains approximately equal amounts of water and fat protons. In out-ofphase conditions, the signal of both will cancel out, leaving the vertebrae completely black. In the case of vertebral pathology, however, the signal will increase and, as such, vertebral metastases (or other lesions) will clearly stand out.

82
Q

(MSK) 53. A 34-year-old man has an MRI of the lumbar spine for lower back pain. This is normal apart from a focal lesion present in the L4 vertebral body. This is reported as a vertebral haemangioma. Which of the following MRI characteristics does this lesion most likely have?

A
  1. F

The variable proportions of vascular and fatty soft-tissue elements influence the MRI appearance of haemangiomas. Lesions with a predominantly fatty matrix show high signal intensity on T1WI, intermediate to high signal intensity on T2WI, and loss of signal on STIR or fat-suppressed T2WI. If the vascular elements predominate, the lesions appear hypointense on T1WI and extremely hyperintense on STIR and T2WI. If MRI is inconclusive, CT may be helpful in identifying the typical pattern of haemangiomatous bone replacement, such as the honeycomb, ‘soap bubble’ or ‘sunburst’ appearance.

83
Q

(Ped) 6 A 15-year-old child with Trisomy 21 presented to the Emergency Department with neck pain and loss of sensation over the occiput. He had fallen awkwardly on his head earlier during a game of touch rugby. What is most likely to be visible on plain radiographs of the neck?

a Sagittal cleft of the vertebral body

b Unilateral facet joint dislocation

c Abnormal odontoid peg formation

d Hypoplastic posterior arch of Cl

e Atlantoaxial subluxation

A

6 Answer E: Atlantoaxial subluxation

Hypoplastic posterior arch of C1 and abnormal odontoid peg can also be found in Down’s syndrome but with the clinical history this child is most likely to have atlantoaxial subluxation.

84
Q

(MSK) 35 A 47-year-old woman sustained a significant flexion injury to the cervical spine as a result of a high-velocity road traffic accident. Her spine was immobilised and she was taken to the local trauma centre for assessment. A lateral cervical spine radiograph revealed 50% anterolisthesis of C4 on C5 and malalignment of the apophyseal joints. There was no suggestion of a rotational component to injury. What type of injury is this?

a Stable unilateral facet dislocation

b Unstable unilateral facet dislocation

c Bilateral facet dislocation

d Jefferson fracture

e Perched facets

A

35 Answer C: Bilateral facet dislocation

This injury is inherently unstable, in contrast to unilateral facet dislocation. Severe flexion forces in combination with distraction result in complete disruption of the facet joints and displacement of the vertebral body by at least 50% relative to the one below it. A significant number are associated with traumatic disc herniation. Initial closed reduction and traction is required, but should be done judiciously as it is possible for a disc herniation to retropulse into the cervical canal.

85
Q

@# (MSK) 37 A middle-aged gentleman who is otherwise well was shown to have coarse, vertically aligned trabeculae on a lumbar spine radiograph. A comprehensive range of blood tests were all normal. A CT of his spine was performed which showed a `pepperpot’ pattern with small dots of high density on axial images. The cortical margins were well preserved. What is the most likely diagnosis?

a Paget’s disease

b Haemangioma

C Metastatic prostate carcinoma

d Multiple myeloma

e Lymphoma

A

37 Answer B: Haemangioma

Haemangiomas are confined to trabecular bone. Their characteristic appearance is due to resorption of trabeculae by enlarged vascular channels and thickening of the remaining trabeculae.

86
Q

(MSK) 60 A 39-year-old male presented with a long history of back pain and was eventually diagnosed with an ependymoma. Where was this lesion most likely to have occurred?

a Within the skull

b Cervical spine

c Thoracic spine

d Lumbar spine

e Sacrum

A

60 Answer B: Cervical spine

Almost two-thirds occur in the cervical spinal cord with one-third of these showing extension to the thoracic cord. They usually extend over multiple segments (three or four on average).?

87
Q

(Ped) 60 Following minor trauma a six-year-old child has cervical spine radiographs. No cervical spine injury is identified and a diagnosis is made of pseudosubluxation. What are the likely radiographic findings?

a A line between the anterior margins of C1 and C3 spinous processes passing within 2 mm of the anterior margin of the C2 spinous process

b 5-mm distance between the odontoid and the anterior arch of the axis

C Anterior displacement of up to 5 mm of the C2 vertebral body on the C3 vertebral body on flexion views

d Increase in the anterior displacement of the C2 vertebral body on the C3 vertebral body on extension views and reduction on flexion views

e 7-mm distance between the odontoid and anterior arch of the axis

A

60 Answer A: A line between the anterior margins of Cl and C3 spinous processes passing within 2 mm of the anterior margin of the C2 spinous process

Pseudosubluxation is normal mobility of the upper cervical spine, particularly at C2/3 and C3/4, which occurs in approximately 40% of children younger than eight years old. It results from immature lax ligaments and horizontally aligned facets. Swischuck’s line, drawn between the anterior margins of the spinous processes of Cl and C3, should pass through or lie within 2 mm of the anterior margin of the spinous process of C2. More than 2 mm is suggestive of pathological subluxation. In pseudosubluxation the anterior displacement occurs in flexion and should reduce with neck extension.

88
Q

63 A 22-year-old gymnast was admitted with signs of right L4 radiculopathy. An MR was performed which revealed disc pathology. What is the most likely site of abnormality on the MR?

a Right L5/S 1 paracentral disc bulge

b Left L5/SI central disc bulge

C Right L4/L5 paracentral disc bulge

d Left L4/L5 paracentral disc bulge

e Right L4/L5 lateral disc bulge

A

63 Answer E: Right L4/L5 lateral disc bulge

Intervertebral disc herniation and degeneration is the most common source of compressive radiculopathy. The lumbar roots emerge from below their respective vertebrae. Different nerves are compressed depending on the location of the disc protrusion. To avoid confusion, nerve roots are designated as exiting' and descending/ traversing’ nerve roots. The exiting nerve is the nerve leaving the spinal canal through the neural foramen below the pedicle of the vertebral body sitting on top of the disc. At each intervertebral disc level, there is usually only one spinal nerve root outside the dural sac in the spinal canal descending behind the intervertebral disc to exit below the pedicle of the vertebral body forming the lower surface of that disc. At the L4-L5 disc, for example, the exiting nerve would be the L4 nerve, which usually leaves the dural sac at about the level of the lower part of the body of L3, descends behind the L3-L4 disc, and exits the spinal canal below the pedicle of L4 through the top of L4-L5 neural foramen. The descending nerve would be the L5 nerve root.

89
Q

64 An 86-year-old woman complains of chronic neck pain. She has had multiple admissions for fractures and is on a bisphosphonate. A skull X-ray is taken. On the lateral film the odontoid peg is seen at the level of McGregor’s line. What is this condition?

a Platybasia

b Craniocaudal disassociation

C Basilar invagination

d Basilar impression

e Normal finding

A

64 Answer C: Basilar invagination

McGregor’s line extends from the posterior limits of the hard palate to the base of the occipital region. The odontoid peg should lie 5 mm below this level. If it is at the level of the McGregor’s line, this is basilar invagination. Causes of basilar invagination are rickets, osteomalacia, fibrous dysplasia and developmental such as Klippel-Feil. Platybasia is flattening of the skull base and it commonly occurs with basilar invagination. The basal angle is greater than 140 degrees in platybasia

90
Q

(MSK) 64 A 42-year-old patient was investigated for acute abdominal pain and had a CT. There were features of inflammatory bowel disease and an incidental finding of a haemangioma in the L1 vertebra. Which part of the vertebra is most likely to be involved?

a Spinous process

b Pars articularis

C Transverse process

d Pedicles

e Vertebral body

A

64 Answer E: Vertebral body

Usually in upper lumbar/lower thoracic spine. Rarely may bulge posterior cortex or extend into spinal canal leading to cord compression.

91
Q

@# 65 A 60-year-old woman was admitted with severe back pain. She had a past medical history of breast cancer and was on tamoxifen. She underwent an MRI scan for further assessment. What feature makes an osteoporotic fracture more likely than metastatic disease?

a Multiple levels affected

b A focal paraspinal mass

C Normal pedicles

d Convex posterior border of the vertebral body

e An epidural mass

A

65 Answer C: Normal pedicles

Features that are suggestive of a bony metastasis are: a bowed posterior border of the vertebral body, abnormal signal in the pedicles, an epidural or paraspinal mass and multiple lesions. Features suggestive of osteoporotic wedge fractures are: low signal on both Ti- and T2-weighted imaging, spared normal marrow signal and multiple compression fractures.

92
Q

66 A 39-year-old male who was being investigated for neck pain was noted on CT to have posterior vertebral scalloping of the C4-7 vertebra. On MRI the cord was expanded at this level by a well defined tumour, which was high signal on T2 and low on T1. There was intense homogeneous enhancement with contrast. What is the most likely diagnosis?

a Meningioma

b Astrocytoma

C Ependymoma

d Lipoma

e Intramedullary haematoma

A

66 Answer C: Ependymoma

Ependymomas are the most common primary spinal cord tumour. They most commonly occur in the cervical region followed by the thoracic region and extend over a large area (average of four vertebral bodies). They are well-defined and central lesions. They often have a low signal rim around the outside due to haemosiderin from repeated bleeds. As they are slow-growing tumours they are ass with bone remodeling, mets to lung, retroperitoneum & LN . There is an 80% 5 Y survival.

93
Q

@# 67 A 60-year-old woman was admitted with immobility and was found to have a raised ESR. The remainder of her blood tests were normal. On T1-weighted MR there was some nodularity along the nerve roots with nodular contrast enhancement along the cauda equina. What is the most likely diagnosis?

a Drop metastasis from ependymoma

b Metastatic breast cancer

C Sarcoid

d Dural sepsis

e Meningioma

A

67 Answer B: Metastatic breast cancer

Drop metastases are most commonly seen in the paediatric population and are most commonly due to PNETs, medulloblastomas, ependymomas, germinomas and pinealoblastomas. In adults metastatic breast and melanoma are common causes. The above description is very typical. Irregularity along the surface can also be seen.

94
Q

68 A 46-year-old male complained of lower back pain and faecal incompetence. On plain film there was a large expansile lyric destructive mass in the sacrum with moderate internal calcification. It was heterogeneous and of low signal on T1 and high on T2. The mass enhanced with contrast. The patient had no other medical history. What is the most likely diagnosis?

a Metastasis

b Plasmacytoma

C Insufficiency fracture

d Teratoma

e Chordoma

A

68 Answer E: Chordoma

This is a typical description of a sacral chordoma. Chordomas originate from embryonic remnants of the notochord. They arise in patients aged between 30 and 70 years of age and are more common in males. The most commonly occur in the sacrum then the clivus. They metastasise to the liver, lung and lymph nodes and tend to recur despite radical surgery.

95
Q

69 A 55-year-old female with progressive lower limb weakness was being investigated by a neurologist. On spinal MR there was a well-defined 1-cm intramedullary spinal mass at D6 level, which was high signal on both TI and T2. A low-signal rim was present on T2-weighted imaging. On T2* there was blooming. No high signal was seen in the cord. What is the most likely diagnosis?

a Ependymoma

b Pilocytic astrocytoma

C Spinal cord cavernous angioma

d AVM

e Metastasis

A

69 Answer C: Spinal cord cavernous angioma

Spinal cavernomas are uncommon, unlike cerebral cavernomas. They typically present in women (M:F = 1:4) between 30 and 50 years of age and are most common in the cervical and thoracic regions. They are intramedullary lesions, which are well defined and are bright on both Ti and T2 as they contain methaemaglobin. Blooming artefact is present because of the presence of haemosiderin from previous bleeds. Little mass effect is seen. A common presentation of spinal cavernomas is with Brown-Sequard syndrome (hemiparaplegia, ipsilateral loss of proprioceptive sensation, hyperesthesia and contralateral loss of pain and temperature). Progression of symptoms is common.

96
Q

(MSK) 25 A young girl dived into a shallow swimming pool and struck the bottom of the pool head first. She was brought to the Emergency Department where she was drowsy and on examination there was a large scalp haematoma and some upper cervical spine tenderness. Which of following would be most appropriate imaging?

a CT head and lateral C-spine radiograph

b Skull radiograph and MRI of neck

C CT head and AP C-spine radiograph

d Skull, AP and lateral C-spine radiographs

e CT head and cervical spine

A

25 Answer E: CT head and cervical spine

Multidetector CT with coronal and sagittal reformatting is now commonplace and the gold standard for cervical spine trauma.

97
Q

28 A six-year-old female was investigated for a neurogenic bladder and persistent leg pain. Clinical examination revealed a hairy patch overlying the lumbosacral region. An MRI showed a bony cleft within the spinal canal and two hemicords from Ti i to LL What is the most likely diagnosis?

a Diastematomyelia

b Sirenomelia

c Myelocystocele

d Dorsal enteric sinus

e Syringohydromyelia

A

28 Answer A: Diastematomyelia

Diastematomyelia is characterised by a developmental sagittal cleft, which splits the spinal cord or filum terminale. The septum is either bony, osteocartilagen- ous or fibrous. This usually occurs between T9 and Si and has a distinct female preponderance. It is associated with other anomalies including Chiari II, tethered cord, neurenteric cyst and dermoids.

98
Q

(MSK) 30 A 44-year-old taxi driver was involved in a 40-mph collision with a stationary car. After assessment he was found to have a stable fracture of his cervical spine and was discharged from hospital. What injury is he most likely to have sustained?

a Posterior arch fracture of the atlas

b Flexion teardrop fracture

C Chance fracture

d Clay-shoveller’s fracture

e Bilateral interfacetal dislocation of C3/4

A

30 Answer D: Clay-shoveller’s fracture

A clay shoveller’s fracture is a stable fracture of a lower cervical vertebra spinous process, usually C7. The mechanism is sudden flexion of the neck combined with a heavy upper body and lower neck muscular contraction. This causes avulsion of the spinous process by the supraspinous ligament. A similar fracture may also occur with direct blows to the spinous process or with occipital trauma causing forced flexion of the neck. The remaining fractures are all potentially unstable.

99
Q

(MSK) 32 A 54-year old lady presented with severe progressive pain in her lumbar spine. As she had a past history of breast cancer (treated with a wide local excision and radiotherapy) a Technetium-99 bone scan was requested. This showed diffuse uptake throughout the lumbar vertebrae and ribs. Which of anatomical sites is most suggestive of metastatic bony disease?

a Vertebral body

b Lamina

C Pedicle

d Sternum

e Superior articular facet

A

32 Answer A: Vertebral body

The most common site for metastatic involvement is the anterior aspect of the vertebral body. Increased uptake in the posterior elements and especially facet joints is more suggestive of degenerative change.

100
Q

39 A 22-year-old male was catapulted from his motorbike. He was admitted unconscious and lateral cervical spine X-ray demonstrated a small triangular fragment of bone antero-inferior to the anterior margin of C4 with widening of the spinous processes at C4-C5. What was likely to have been the mechanism of injury?

a Vertical compression

b Flexion with rotation

C Hyperextension

d Hyperflexion

e Lateral flexion/shearing

A

39 Answer D: Hyperflexion

Hyperflexion injury of the spine accounts for 46-79% of all cervical spine injuries. The anteroinferior teardrop fracture is the most severe and unstable injury of the spine. The widening of the posterior elements indicates disruption of the posterior ligament complex. It is an unstable injury, which results in ligamentous damage and spinal cord compression. Typically, these occur at C6 to C8. Hyperflexion injuries are the most common injury accounting for between 50 and 75% of injuries. The anterior height of the vertebral body is reduced with anterior reduction in disc space and there is diffuse prevertebral soft-tissue swelling. Extension teardrop fractures occur at the anteroinferior corner of C2 and this may occur in isolation or with a hangman’s fracture. Here there is widening of the anterior disc space.

101
Q

(Ped) 40 A 12-year-old girl attended the Accident and Emergency Department following a fall from a bicycle. On examination she had some neck tenderness and was noted to have a short neck and a low hairline. Plain cervical spine radiographs were performed. No fracture was identified, but there was apparent fusion of the C2 and C3 vertebral bodies. Which of the following is the most likely diagnosis?

a Turner’s syndrome

b Klippel-Feil syndrome

C Sprengel’s deformity

d Cleidocranial dysplasia

e Previous fracture at C2/C3

A

40 Answer B: Klippel-Feil syndrome

Klippel-Feil syndrome is caused by failure of normal segmentation of the cervical vertebrae. Features include a short neck, low hairline and reduced mobility of the upper spine. Any of the cervical vertebrae can be involved but fusion of the C2/C3 vertebrae is the most common.

102
Q

@# (MSK) 41 A 19-year-old female presented with lower back pain and a lumbosacral X-ray showed an expansile lyric lesion in the right sacrum. The margins were well defined and there was no softtissue mass visible. No other lesions were suspected. What is the most likely diagnosis?

a Multiple myeloma

b Osteoid osteoma

C Chordoma

d Giant cell tumour

e Aneurysmal bone cyst

A

41 Answer D: Giant cell tumour (GCT)

GCTs are characteristically well defined with a non-sclerotic margin and are most frequently seen in young adults aged between 20 and 40 years. Most GCTs occur in the long bones, but a number do occur in the spine where they tend to affect younger patients and to be three to four times more common in the sacrum than rest of spine.

103
Q

(MSK) 46 A man attended a physiotherapy clinic for treatment of long-standing lower back pain. There is no history of trauma and the physiotherapist noted that the patient was systemically well. As he was not improving, a radiograph was requested, which showed a lesion in L4. Which feature would support the diagnosis of an osteoblastoma over another diagnosis?

a Location in the posterior elements

b Patient age under 30

c Absence of a sclerotic rim

d Less than 2-cm diameter

e Cortical disruption

A

46 Answer A: Location in the posterior elements

An osteoblastoma is histologically identical to an osteoid osteoma but usually larger and the classical description is of a well-defined expansile lesion in the posterior elements of the spine. Around 80% of cases present before 30 years of age and a sclerotic rim is usually present. There is, however, a wide variety of possible appearances including cortical thinning and a soft-tissue mass which can resemble a malignant process.

104
Q

(Ped) 55 A teenage boy suffers from thoracic pain aggravated by exertion. On examination he is found to have a thoracic kyphosis. He is thought to have Scheuermann’s disease. Radiographs of the thoracic and lumbar spine are performed. Which of the following would be the most likely findings?

a Multiple vertebral fusion abnormalities

b Loss of anterior vertebral height and Schmorl’s nodes

c Loss of posterior vertebral height

d Calcification of the anterior spinal ligament

e Multiple vertebral segmentation abnormalities

A

55 Answer B: Loss of anterior vertebral height and Schmorl’s nodes

Scheuermann’s disease or adolescent kyphosis is the second most common paediatric spinal deformity, which has its onset at puberty. In the forward flexed position there is abrupt angulation in the mid to lower thoracic region. Additional radiographic features include disc space narrowing, end plate irregularities and kyphosis >40 degrees.

105
Q

60 A middle-aged male presented with increasing paraesthesia in the right leg. An X-ray of his lumbar spine and views of his leg were unremarkable. An MRI was performed which showed a lesion displacing the thecal sac away from the bony spinal canal. Which of the following would you not include in the differential diagnosis?

a Prolapsed disc

b Epidural metastases

C Neurofibroma

d Ependymoma

e Meningioma

A

60 Answer D: Ependymoma

The lesion described is extradural in location.

Extradural masses arise from outside the spinal dura and may originate from the vertebral body, disc and adjacent soft tissues. On MR dura may be seen draped over the mass, and sometimes epidural fat may be seen capping the lesion.

Intradural, extramedullary lesions arise inside the thecal sac, but outside the cord and originate from the nerve roots, meninges and CSF spaces.

On MRI, intradural lesions are clearly delineated by CSF and the cord is deviated away from the lesion. The subarachnoid space is enlarged up to the mass.

Intramedullary masses arise inside the spinal cord and originate from the cord parenchyma or pia. On MRI there is diffuse enlargement of the cord with gradual effacement of the subarachnoid space.

106
Q

@# (MSK) 60 A 29-year-old male presented with difficulty walking and was found to have a complex sensory deficit. After investigation he was found to have an astrocytoma of the spinal cord. Which area is most likely to be involved?

a Brainstem

b Cervical spine

C Thoracic spine

d Lumbar spine

e Sacrum

A

60 Answer C: Thoracic spine

Almost two-thirds occur in the thoracic spinal cord, although half involve the cervical cord as they usually extend over a long region of cord (approximately seven segments on average).

107
Q

@# 61 A six-year-old child presented with gait problems and neurologic deficit. An MRI of the brain and whole spine was performed. There was a diffuse abnormality in the upper thoracic cord extending over approximately five vertebral levels. The lesion appeared intramedullary in location and was hypo- isointense on Ti and hyperintense on T2-weighted images with enhancement on Ti following IV Gadolinium. A syrinx was seen more superiorly. What is the most likely diagnosis?

a Ependymoma

b Transverse myelitis

C Astrocytoma

d Metastases

e Haemangioblastoma

A

61 Answer C: Astrocytoma

The commonest intramedullary lesions are astrocytoma and ependymoma. Astrocytomas are the most common cord tumour in children and the cervical cord is the commonest location followed by the thoracic cord. Multisegmental involvement is common and they are often associated with a syrinx and cysts. Despite being low grade they tend to enhance strongly with contrast.

Ependymomas are more common in adults and occur in the conus medullaris and filum terminale. They are generally slow growing and cause vertebral body scalloping. Cysts and haemorrhage are common.

108
Q

@# 62 A 26-year-old male with haemophilia developed sudden onset cauda equina syndrome and an emergency MRI of the lumbar spine was performed. A large fusiform posterior extradural mass extending from L1 to L5 was present. It was hypointense on T2-weighted images. What is the most likely diagnosis?

a Epidural abscess

b Metastasis

c Arachnoid cyst

d Neurofibroma

e Haematoma

A

62 Answer E: Haematoma

Spinal haematomas occur due to trauma, following a procedure (typically laminectomy or epidurals), during childbirth or spontaneously, particularly in those with bleeding disorders. They are usually due to venous bleeds and are typically posterior fusiform masses. They can result in a cauda equine syndrome and require prompt neurosurgical evacuation.

109
Q

64 A middle-aged woman had a three-year history of change in sensation in her lower legs. On MRI of her whole spine there was diffuse cord expansion in the thoracic region with cystic areas. The abnormality was isointense on Ti and hyperintense on T2 with multiple flow voids and peripheral enhancement. Bilateral renal masses were just visible on the edge of the MR images. What is the most likely diagnosis?

a Renal cyst with astrocytoma

b Von Hippel-Lindau syndrome with haemangioblastoma and renal cell carcinomas

C Neurofibromatosis with renal cysts and spinal neurofibromas

d Metastatic breast cancer

e Arteriovenous malformation with myelolipomas

A

64 Answer B: Von Hippel-Lindau syndrome with haemangioblastoma and renal cell carcinomas

A third of all spinal haemangioblastomas occur in patients with VHL. They are the third most common intramedullary neoplasm. They are usually intramedullary but can be extramedullary (25%) and occur most commonly in the cervical (40%) and thoracic (50%) regions. Patients present with a prolonged history of sensory and motor symptoms. They are highly vascular lesions, which are isointense on Ti, high signal on T2 with multiple signal voids and bright enhancement with contrast. Low signal from haemosiderin is often seen on T2*.

110
Q

66 A newborn girl was found at birth to have a hairy naevus overlying the sacral region and an imperforate anus. She underwent an ultrasound of her spine. The conus was found to lie at L4 and the filum terminale measured 4 mm in diameter. What is the most likely diagnosis?

a Caudal regression syndrome

b Tethered cord

C Diastematomyelia

d VACTRL

e Spinal bifida occulta

A

66 Answer B: Tethered cord

Tethered cord usually presents in children with bowel and bladder dysfunction and lower limb neurology. It is slightly more common in females. Fifty per cent of patients have a hairy patch overlying the lower back. It can be associated with filum lipoma, imperforate anus, filum cysts and diastematomyelia. The main features are low-lying cord and thickened filum terminale (greater than 2 mm). Patients are treated with a decompressive laminectomy. At birth the corms medullaris should lie at L2/3 and at L1/2 at three months.

111
Q

67 A 35-year-old female was being investigated for bilateral lower limb neurology. On plain film Xray she was found to have bilateral pars defect with a 50% spondylolisthesis. MR was performed which did not demonstrate any neural foraminal narrowing or nerve root impingement. What investigation you suggest was performed next?

a Axial T2-weighted brain

b Coronal FLAIR brain

C Nerve conduction studies

d Psychiatric opinion

e Flexion extension views of lumbar spine

A

67 Answer E: Flexion extension views of lumbar spine

Pars defect is a fracture through the pars interarticularis. These are due either to a stress fracture, congenital or secondary to other condition; for example, tumours, osteomalacia or Paget’s disease. Spondylolysis commonly occur at the L5 vertebra. Neurology typically occurs after a degree of spondylolisthesis. If on imaging no nerve root impingement is seen but the history is typical, this might be because the slip is unstable and so flexion extension films should be performed.

112
Q

@# 68 An 18-year-old boy was involved in a road traffic accident. He had multiple injuries and one month after his accident he still had a right Ti nerve root palsy. MRI of the brachial plexus was normal. A further MRI study of his cervicothoracic spine was performed which showed an absent right Ti nerve root. No conjoint roots were seen at C7 or T2. There was a small, well-defined area of CSF signal at the right Ti neural exit foramina. What is the most likely diagnosis?

a Lateral myelomeningocele

b Traumatic nerve root avulsion

C Tarlov cyst

d Neurogenic cyst

e Synovial cyst from the facet joint

A

68 Answer B: Traumatic nerve root avulsion

The right Ti nerve root has been avulsed. This most commonly occurs in the cervical region following severe acute traction on the upper limb such as a fall from a motorbike. Imaging typically demonstrates an absent nerve root within the neural foramina and a pseudomeningocele. If patients are not able to have an MRI, a CT myelogram could be performed.

113
Q

@# 69 A 55-year-old woman was being investigated for right lower limb weakness. On MRI there was a well-defined high signal cystic area in the right Si lateral recess. There was expansion of the right Si neural canal with bone scalloping. Nerve roots are visualised along the wall of the cyst. What is the most likely diagnosis?

a Tarlov cyst

b Myelomeningocele

C Schwannoma of right Si nerve root

d Pilocytic astrocytoma

e Haematoma

A

69 Answer A: Tarlov cyst

A Tarlov cyst is a perineural cyst arising from the nerve root. They most commonly occur in the sacral region and can cause bone scalloping from pressure effects. It is postulated that there is a ball valve effect so CSF flows into the cyst with arterial pulsations. The spinal nerves may be visualised either in the wall of the cyst or the cyst itself. They occur in 5 % of the population and are more common in women. They are usually asymptomatic but may cause symptoms such as bladder and bowel dysfunction or lower motor or sensory abnormalities.

114
Q

@# (ped) 2 The paediatric team sees a neonate with respiratory distress, bradycardia and poor swallowing. Following imaging investigation the child was found to have a small posterior fossa and dysgenesis of the hindbrain. The fourth ventricle and hindbrain are displaced caudally and the tonsils and vermis are herniating through the foramen magnum. What further CNS abnormalities may be present?

a A funnel-shaped posterior fossa

b Klippel-Feil deformity

C Basilar impression

d Herniation of the cerebellar tonsils

e Lumbar myelomeningocele

A

2 Answer E: Lumbar myelomeningocele

The child has Arnold Chiari malformation (Chiari II malformation), which has the above characteristic features. It is associated with lumbar myelomeningocele in >95% of cases and syringohydromyelia. In addition it is associated with the following supratentorial anomalies:
* dysgenesis of corpus callosum (80-85%)
* obstructive hydrocephalus secondary to closure of myelomeningocele (50-98%)
* absence of septum pellucidum (40%)
* excessive cortical gyration.
It is notably not associated with basilar impression, Cl assimilation and KlippelFeil deformity

115
Q

(MSK) 4 A 50-year-old man presented with back pain and subsequent imaging demonstrated flowing ossification of six thoracic vertebral bodies. There was no evidence of ankylosing spondylitis and a diagnosis of diffuse idiopathic skeletal hyperostosis (DISH) was made. What further feature would be most likely on imaging?

a Ossification of patellar ligament

b Sclerotic sacroiliac joints

C Talar osteophytes

d Fractures of the iliac crest

e Osteophytes of single vertebral bodies

A

4 Answer A: Ossification of patellar ligament

DISH is also known as Forestier disease or ankylosing hyperostosis and is an ossifying diathesis with a proliferation of entheses (bony growths at tendon and ligament insertions). DISH principally affects the spine, pelvis and extremities.

The differential diagnosis includes: fluorosis, acromegaly, ankylosing spondylitis and intervertebral osteochondrosis.

The features are ossification of patellar ligament, heel spurs, whiskering of the iliac crest and flowing ossification of at least four contiguous vertebral bodies

116
Q

(Ped) 48 A 14-year-old girl is referred to the orthopaedic clinic with scoliosis. AP radiographs of the spine are taken. Two lines are drawn and the Cobb angle is measured between them to confirm the diagnosis. Where should the lines be drawn to correctly measure the Cobb angle?

a From the centre of the apical vertebra to the centre of the superior end vertebra and from the centre of the apical vertebra to the centre of the inferior end vertebra

b Tangential to the superior end plate of the apical vertebra and tangential to the inferior endplate of the apical vertebra

c Tangential to the inferior end plate of the superior end vertebra and tangential to the superior end plate of the inferior end vertebra

d Tangential to the superior end plate of the superior end vertebra and tangential to the inferior end plate of the inferior end vertebra

e From the centre of the apical vertebra to the superior end plate of the superior end vertebra and from the centre of the apical vertebra to the inferior end plate of inferior end vertebra

A

48 Answer D: Tangential to the superior end plate of the superior end vertebra and tangential to the inferior end plate of the inferior end vertebra

The Cobb angle is the most commonly used method of measuring the scoliotic curve on an AP view. It divides severity of curvature into seven categories and is used to aid selection of patients for surgical treatment and monitoring results of treatment. The vertebrae used in measurement must be noted and the same vertebrae used in subsequent measurements to allow comparison. The RisserFerguson method, which is used less commonly, is measured as described in answer A. The result is not comparable with the Cobb angle.

117
Q

59 A 29-year-old heroin addict presented with sudden onset left-sided leg pain. An X-ray of the lumbar spine was performed followed by an MRI, which confirmed the clinical suspicion of infective spondylitis. What features would you expect on the MRI?

a Narrowing of the disc space with high signal in the adjacent vertebral bodies on T1- weighted images

b Narrowing of the disc space with high signal in the adjacent vertebral bodies on T2- weighted images

c Widening of the disc space with high signal in the adjacent vertebral bodies on T1-weighted images

d Widening of the disc space with high signal in the adjacent vertebral bodies on T2- weighted images

e None of the above

A

59 Answer B: Narrowing of the disc space with high signal in the adjacent vertebral bodies on T2-weighted images

Early diagnosis is crucial in the management of infective spondylitis. Staphylococcus aureus is the most common pathogen. Imaging abnormalities on X-rays are often subtle and detected late in the disease. MR findings are characteristic and consist of reduced disc space and low signal on Ti and high signal on T2 in the adjacent vertebral bodies (reflecting increased extracellular fluid in the bone marrow). Subligamentous or epidural soft-tissue masses maybe seen.

118
Q

@# 60 A 20-year-old man had progressive upper and lower limb weakness, worse in the lower limbs. On MR there was widening of the spinal canal with posterior vertebral scalloping between D3 and D7. On Ti- and T2-weighted imaging a well-defined high-intensity mass was present anterior to the spinal cord with atrophy of the cord at this level. The CSF space was slightly expanded immediately superior to the mass. No high signal was present in the cord on T2. What is the most likely diagnosis?

a Epidural abscess

b Epidural haematoma

C Neuroma

d Neurogenic cyst

e Meningioma

A

60 Answer D: Neurogenic cyst

The description above describes a chronic process resulting in bone remodelling and atrophy of the cord. Thus an epidural abscess and haematoma are unlikely. This mass is extramedullary but intradural. Although meningiomas and neuromas are intradural they are isointense on Ti hence the best answer is a neurogenic cyst. Neurogenic cyst is an intradural mass, which is commonly seen within the cervical and thoracic region. They are associated with other spinal abnormalities such as diastematomyelia and Klippel-Feil syndrome.

119
Q

61 A 54-year-old female was scanned following insidious onset of bilateral leg weakness. MRI of the thoracolumbar spine revealed gibbus deformity centred at T10. Destructive changes were seen involving both anterior and posterior elements above and below this level with loss of vertebral body and intervertebral disc height. An epidural abscess was seen tracking down to L1 level. What is the most likely diagnosis?

a Brucellosis

b Tuberculosis

C Pyogenic spondylitis

d Actinonnycosis

e Herpes simplex virus

A

61 Answer B: Tuberculosis

Tuberculous spondylitis commonly occurs with an insidious onset in the thoracolumbar region with contiguous involvement of multiple vertebrae. Spread is usually via the haematogenous route and the vertebral bodies are involved more commonly than the posterior elements. Destruction of the vertebral body leads to vertebra plana in children and gibbus deformity in adults.

Pyogenic spondylitis is the major differential which occurs more rapidly with destruction, little new bone formation and usually sparing of the posterior elements.

In brucellosis, lower lumbar involvement is common and bone destruction is associated with sclerosis. Epidural extension is minimal.

120
Q

@# 62 Following surgery for a herniated L4/L5 disc a 66-year-old obese patient had little symptomatic relief. An MRI scan performed in the second postoperative week revealed extradural soft-tissue material within the spinal canal, which demonstrated little enhancement following contrast. Nerve root enhancement was striking. What is the most likely diagnosis?

a Arachnoiditis

b Epidural haematoma

c Residual disc material

d Epidural fibrosis

e Neuritis

A

62 Answer C: Residual disc material

Appearances of the post-operative spine can be challenging.

In the early postoperative period, persistent symptoms are usually due to epidural haematoma, retained fragment or recurrent disc.

In the subacute and chronic stage the differential is mainly between a disc and epidural fibrosis.

Osseous abnormalities depend on the specific surgical procedure.

Epidural fibrosis occurs commonly, enhances with contrast administration and the degree of enhancement varies with time since the operation, enhancing most strongly within a year following surgery.

Neuritis, identified as intrathecal enhancement of nerve roots, is seen in approximately 20% of symptomatic patients.

Early postoperative root enhancement is common in asymptomatic patients and is considered significant only if it persists beyond at least six to eight months.

121
Q

63 A 46 year old undergoing a CT of the abdomen and pelvis for small bowel pathology was noted to have an abnormal-looking T10 vertebral body. This demonstrated some expansion with internal lucencies and areas of coarse trabeculation; the posterior elements were spared. An X-ray of the spine demonstrated vertical striations at this level. What is the most likely diagnosis?

a Haemangioma

b Osteoid osteoma

C Giant cell tumour

d Osteoblastoma

eABC

A

63 Answer A: Haemangioma

Vertebral haemangiomas are commonly incidental findings. Pain may occur and these become symptomatic if pathologic compression fracture or haemorrhage occurs. Plain X-rays demonstrate trabecular thickening and vertical striations while a `polka dot’ appearance is seen on CT On MRI the signal may be variable, although they are usually high on Ti and T2. The major differential on MRI is focal fatty replacement of the marrow and the distinction may be made with fat suppressed sequences. Osteoid osteoma, osteoblastoma and ABC all involve the posterior elements

122
Q

64 A 35-year-old woman with a known spondylolysis and 50% spondylolisthesis presented with new onset lower limb weakness. She had an episode of vertigo six months previously, which had resolved. MRI of the spine demonstrated the spondylolisthesis and bilateral pars defects, but no cause for the symptoms was found. On flexion extension films the spondylolisthesis was found to be stable. What would be the next investigation you would suggest to the clinician?

a Lumbar puncture

b CT brain

C MRI brain

d Nerve conduction studies

e Repeat the MR in six months

A

64 Answer C: MRI brain

The MRI and flexion lumbar spine films confirm that the spondylolisthesis is stable and not impinging on the nerve roots. There are now two episodes of neurology affecting different parts of the body, spaced in time, and so this patient may have multiple sclerosis. The next best test is an MRI of the brain.

123
Q

65 A 12-year-old with a webbed neck, low hairline and a decreased range of neck movement underwent a CT of the cervicothoracic spine for surgical planning. Clinically, a diagnosis of Klippel-Feil syndrome had been made. Which of the following would not be an expected feature on the scan?

a Fusion of two or more vertebrae

b Raised and rotated scapula

C Paravertebral ossification

d Omovertebral bar

e Spinal stenosis

A

65 Answer C: Paravertebral ossification

A short neck, reduced cervical movement and low posterior hair line are typical findings in Klippel- Feil syndrome, which usually affects the upper cervical region with fusion of vertebral bodies and posterior column. Other features are scoliosis, hemivertebrae, atlantoaxial fusion, rib fusion and ear abnormalities. Sprengel’s deformity is often seen with this condition in 25 % of cases and results in rotation and elevation of the scapula due to an omovertebral connection.

124
Q

66 A 38-year-old male had a long history of back stiffness and restricted movement. Spinal X-rays followed by a CT of the thoracolumbar spine revealed flowing calcifications and ossifications along the right anterolateral aspect of the T7-T11 vertebral bodies. Disc spaces appeared preserved. What is the most likely diagnosis?

a Ankylosing spondylitis

b Diffuse idiopathic skeletal hyperostosis

C Reiter’s disease

d Psoriatic arthritis

e Primary osteoarthritis

A

66 Answer B: Diffuse idiopathic skeletal hyperostosis

DISH is characterised by diffuse flowing ossification at the site of ligamentous and tendinous insertion points. Patients are normally over 50 and it is more common in men. Pain, tenderness and restricted movement are all common complaints. It most commonly affects the mid and lower thoracic regions and extends over at least four contiguous vertebral bodies. The pelvis can be affected with ossification of iliolumbar & sacrotuberous ligaments.

125
Q

67 A 60-year-old man was being investigated for progressive bilateral leg weakness. An MR lumbar spine was performed on which multiple tortuous tubes with signal voids on the posterior aspect of the cauda equine were visible on both T1- and T2-weighted images. Subtle bumps were seen along the surface of the cord. The conus was expanded with high signal within it on T2. What is the likely diagnosis?

a Epidural haematoma

b Epidural abscess

C Cavernoma

d Spinal dural arteriovenous fistula

e Spinal cord arteriovenous malformation

A

67 Answer D: Spinal dural arteriovenous fistula

There are four types of spinal AVM: spinal dural arteriovenous fistula (Type 1), spinal cord AVM (Type 2 and 3) and spinal cord AV fistulas (Type 4). Spinal AVMs present either with progressive sensory and motor loss or acutely following a sudden bleed which has a significant mortality. Haemosiderin may be seen in the subarachnoid spaces reflecting previous bleeds. Spinal AVMs may acutely thrombose and high signal within the cord is very common. Spinal cord arteriovenous malformations are intramedullary while spinal dural and spinal cord arteriovenous malformations are extramed but intradural. Treatment options are embolisation, surgery or both.

126
Q
  1. A 40-year-old housewife presents with severe left L4 radiculopathy. 1 year ago she had a L4/5 discectomy. A gadolinium-enhanced MRI of the lumbar spine was performed. Post surgical fibrosis and epidural scar is diagnosed on T1-enhanced images by?

(a) An enhancing epidural tissue at L4/5 compressing on left L4 nerve root

(b) A non-enhancing epidural tissue at L4/5 compressing the left L4 nerve root

(c) An enhancing mass within the spinal canal at L4/5 compressing left L4 root

(d) A non-enhancing mass in the spinal canal at L4/5 compressing left L4 root

(e) An enhancing mass at L5/S1 compressing left L4 root

A
  1. (a) An enhancing epidural tissue at L4/5 compressing on left L4 nerve root

Scar tissue seen after previous disc resection shows enhancement with gadolinium, while a recurrent or sequestrated disc is unlikely to.

127
Q

(MSK) 9. The following MRI features favour spinal mets fracture rather than an osteoporotic collapse: (T/F)

(a) Convex posterior border of the vertebral body.

(b) Diffuse paraspinal mass.

(c) Abnormal signal in the neural arch.

(d) Low signal intensity band on T1W MRI.

(e) Areas of normal marrow signal vertebral body.

A

Answer:

(a) Correct
(b) Not correct
(c) Correct
(d) Not correct
(e) Not correct

Explanation:

Diffuse paraspinal mass is not helpful in differentiating between metastatic and osteoporotic collapse. However focal paraspinal mass is seen more frequently with metastatic collapse. Low signal intensity band on T1WI and T2WI is more common in osteoporotic collapse (93%) than in metastatic collapse (44%). Spared normal marrow signal in a vertebral body is highly suggestive of osteoporotic collapse.

128
Q

(MSK) 20. Which of the following are correct regarding musculoskeletal trauma. (T/F)

(a) Sternal fracture an indirect sign of thoracic spine injury

(b) Atlanto-occipital dislocation is prequently fatal

(c) Laceration of the dura is commonly associated with lumbar spine fracture

(d) A scapholunate angle of more than 80% is suggestive of scapholunate dissociation

(e) Epiphyseal plate fracture (Salter-Harris type I) are common in nonaccidental injury in children.

A

Answers:

(a) Correct
(b) Correct
(c) Not correct
(d) Correct
(e) Not correct

Explanation:

Laceration of the dura is not associated with lumbar spine fracture. It is more common in patients with a burst fracture that is associated with neurological deficit and laminae fracture. The epiphyseal plate injuries are rarely encountered in non-accidental trauma.

129
Q

(MSK) 21. Which of the following are correct with regards to the cervical spine: (T/F)

(a) For the erect lateral view of the cervical spine the central beam is directed horizontally to the centre of C3 vertebra.

(b) The space between the odontoid process and the anterior arch of the atlas (atlanto-dens interval) should not exceed 3 mm in adults

(c) A Jefferson fracture is unstable

(d) A swimmer’s view can be used for better demonstration of the C1/C2 junction.

(e) A hangman’s fracture is usually secondary to a hyper-flexion injury

A

Answers:

(a) Not correct
(b) Correct
(c) Correct
(d) Not correct
(e) Not correct

Explanation:

For the erect lateral view the central beam is directed to the centre of C4. A swimmer’s view is used for better visualization of C7/C8/T1. Hangman’s fracture is secondary to hyperextension resulting in bilateral fractures of pedicles of C2. It is unstable injury and accounts for 4-7% of all spinal fractures.

130
Q
  1. Regarding diffuse idiopathic skeletal hyperostosis which of the following are correct: (T/F)

(a) Sacroiliac joints are usually involved when the lumbar spine is.

(b) Highest incidence in the 3rd to 5th decades.

(c) Most commonly involves the cervical spine.

(d) Extraspinal ligamentous hyperostosis is a feature.

(e) In the thoracic spine the hyperostoticchanges are more prominent on the right.

A

Answers:

(a) Not correct
(b) Not correct
(c) Not correct
(d) Correct
(e) Correct

Explanation:

Diffuse idiopathic skeletal hyperostosis is a common condition characterised by bone proliferation at sites of tendinous and ligamentous insertion of the spine affecting elderly individuals.It commonly affects the elderly more than 50years of age with a slight male predominance. It most commonly involves the lower thoracic spine. Sacroiliac joints are not involved. However, the appearance of spine may be similar to ankylosing spondylitis.

131
Q
  1. Which of the following are correct regarding trauma of the neck? (T/F)

(a) The most common site of vertebral artery injury in blunt trauma to the neck is at the C5/C6 level.

(b) Multiple levels are involved in 20% of fractures of the cervical spine.

(c) In blunt neck trauma, dissection of the extracranial carotid artery is asymptomatic in 10% of cases.

(d) Fracture of the odontoid peg most commonly involves the tip.

(e) Barium swallow has poor sensitivity for pharyngoesophageal injury is blunt neck trauma.

A

Answers:

(a) Not correct
(b) Correct
(c) Not correct
(d) Not correct
(e) Not correct

Explanation:

Vertebral artery injury is usually due to stretching and occurs most frequently at the C1-C2 level. Vertebral artery injuries are more common than carotid artery injuries in blunt neck trauma. Dissection of the extracranial carotid artery is asymptomatic at presentation in 50% of the cases. Diagnosis may be delayed.

132
Q
  1. Which of the following are correct regarding Klippel-Feil syndrome? (T/F)

(a) Vertebral fusion involves the bodies and neural arches.

(b) Coarctation is a feature.

(c) It is associated with Sprengel’s shoulder deformity.

(d) Conductive hearing loss is a feature.

(e) Lymphoedema is a feature.

A

Answers:

(a) Correct
(b) Correct
(c) Correct
(d) Correct
(e) Correct

Explanation:

Klippel Feil syndrome is also associated with syringomyelia, platybasia, clubfoot, hemivertebrae and congenitally heart diseases. Vertebral fusion may involve the craniovertebral junction and cervicothoracic junctions.

133
Q
  1. Which of the following are correct regarding fractures of the cervical spine? (T/F)

(a) Flexion teardrop fractures involve the superior endplate of the vertebral body.

(b) Hangman’s fracture is a bilateral fracture of the neural arches of C2.

(c) Unilateral interfacet joint dislocation is a stable injury.

(d) Clay shoveler’s fracture is stable.

(e) In bilateral interfacetal dislocation, there is anterior translocation of the involved vertebra by at least 50% of the diameter of the subjacent vertebrae.

A

Answers:

(a) Not correct
(b) Correct
(c) Correct
(d) Correct
(e) Correct

Explanation:

Flexion teardrop fractures involve avulsion of the anterior inferior corner, usually of the C2 vertebral body with displacement of the involved vertebral body into the spinal column. There is destruction of all soft tissue structures with an associated acute anterior cervical cord syndrome.